Med surg exam 1- cardiovascular

Pataasin ang iyong marka sa homework at exams ngayon gamit ang Quizwiz!

B

A 54 year old cardiac pt with iron deficieny anemia tells the nurse that she prepared low cholesterol food for her family and doesn't eat red meats. An appropriate goal for this pt would be to increaser her intake of: a. eggs and fish b. legumes and whole grains cereals and breads c. milk and milk products d. nuts and cornmeal

A. stable angina is relieved by rest not unstable

A 64 year old pt has unstable angina. Which symptom is she least likely to report? a. chest pain relieved by rest b. chest pain resulting from exertion c. chest pain resulting from emotional stress d. chest pain that is not relieved by nitroglycerin

3. A CT scan will determine if the client is having a stroke or has a brain tumor or another neurological disorder. If a CVA is diagnosed, the CT scan can determine if it is a hemorrhagic or ischemic accident and guide treatment. 1.The drug rt-PA may be administered, but a cerebrovascular accident (CVA) must be verified by diagnostic tests prior to administering it. rt-PA helps dissolve a blood clot, and it may be administered if an ischemic CVA is verified; rt-PA is not given if the client is experiencing a hemorrhagic stroke. 2.Teaching is important to help prevent another CVA, but it is not the priority intervention on admission to the emergency department. Slurred speech indicates problems that may interfere with teaching. 4.The client may be referred for speech deficits and/or swallowing difficulty, but referrals are not priority in the emergency department.

A 78-year-old client is admitted to the emergency department with numbness and weakness of the left arm and slurred speech. Which nursing intervention is priority? 1. Prepare to administer recombinant tissue plasminogen activator (rt-PA). 2. Discuss the precipitating factors that caused the symptoms. 3. Schedule for a STAT computed tomography (CT) scan of the head. 4. Notify the speech pathologist for an emergency consult.

1. The client is at risk for increased intracranial pressure whenever performing the Valsalva maneuver, which will occur when straining during defecation. Therefore, stool softeners would be appropriate. 2. Coughing increases intracranial pressure and is discouraged for any client who has had a craniotomy. The client is encouraged to turn and breathe deeply, but not to cough. 2. Monitoring the neurological status is appropriate for this client, but it should be done much more frequently than every shift. 3. Dopamine is used to increase blood pressure or to maintain renal perfusion, and a BP of 160/90 is too high for this client.

A client diagnosed with a subarachnoid hemorrhage has undergone a craniotomy for repair of a ruptured aneurysm. Which intervention will the intensive care nurse implement? 1. Administer a stool softener b.i.d. 2. Encourage the client to cough hourly. 3. Monitor neurological status every shift. 4. Maintain the dopamine drip to keep BP at 160/90.

1. Dyspnea on exertion (DOE) is typically the earliest manifestation of mitral valve stenosis. 2. Jugular vein distention (JVD) and 3+ pedal edema are signs/symptoms of right-sided heart failure and indicate worsening of the mitral valve stenosis. These signs would not be expected in a client with early manifestations of mitral valve stenosis. 3. Chest pain rarely occurs with mitral valve stenosis. 4. An enlarged liver and edematous abdomen are late signs of right-sided heart failure that can occur with long-term untreated mitral valve stenosis.

A client is being seen in the clinic to R/O mitral valve stenosis. Which assessment data would be most significant? 1. The client complains of shortness of breath when walking. 2. The client has jugular vein distention and 3+ pedal edema. 3. The client complains of chest pain after eating a large meal. 4. The client's liver is enlarged and the abdomen is edematous.

A these are findings associated with infective endocarditis B.Pt with pericarditis would report chest pain C. Pt with myocarditis would report a rapid heart rate D.Pt with rheumatic endocarditis would report joint pain

A nurse assessing a pt who has splinter hemorrhages of the nail beds and reports a fever. What is the disorder? A. infective endocarditis B. pericarditis C. myocarditis D.rheumatic endocarditis

C.Metoprolol masks hypoglycemia A.AE of psyllium is not hypoglycemia B. skim milk increases glucose D.grapefruit juice increases blood sugar

A nurse at an urgent care clinic is getting a Hx of a client with DM and a new diagnosis of HTN. This is the second time in two weeks that the client has experienced hypoglycemia. What data should the nurse report to the provider? A.takes psyllium daily as a fiber laxative B.drinks skim milk daily as a bedtime snack C.takes metoprolol daily after meals D. drinks grapefruit juice daily with breakfast

A. manifestation of right sided failure B. fatigue is a manifestation of right sided failure E. edema is a manifestation of right sided failure C. normal of tachycradia is expected with right sided failure D. pleural friction manifestation of pneumonia

A nurse educator is reviewing expected findings in a client who has right sided valvular heart disease. Which of the following findings should the nurse include is the discussion? (select all that apply) A. dyspnea B.client reports fatigue C. bradycardia D. pleural friction rub E. peripheral edema

D. throat culture would reveal streptococcus which can lead to rheumatic fever A. arterial blood gases are used to measure respiratory status B. blood albumin measures nutrition status C.liver enzymes monitor the clients response to antibiotics therapy

A nurse is admitting a client who had suspected rheumatic endocarditis. The nurse should expect a Rx for which of the following lab tests to assist in confirming the Dx? A. arterial blood gases B. serum albumin C. liver enzymes D. throat culture

B. ABCs A. not the highest priority C. not the highest priority D. not the highest priority

A nurse is caring for a client who has HF and reports increased SOB. Which of the following actions should the nurse take first ? A. Obtain a Wt B.Assist the client to high Fowler's position C. auscultate lung sounds D. check O2 sat

B. Surgery is indicated for older adults whos disease is affecting ADLs A. improves blood flow through a valve C. can improve quality, but not prolong it D. can not reverse damage

A nurse is caring for an older adult who is undergoing a percutaneous balloon valvuloplasty. The pt family asks the nurse to explain the expected outcomes. Which of the following responses should the nurse give? A. " this will improve blood flow of the coronary arts" B. "this will assist with the ability to perform ADLs" C. "this will prolong the lifespan of living with this valve disorder" D. " this will reverse the effects to the damaged area"

B. anticoagulant therapy with warfarin is necessary for a pt following the placement of a mechanical valve. PT time should be checked A. activity limitation for 6weeks C.antibiotic therapy is recommended after dental work D. should be limiting Na

A nurse is completing discharge teaching with a client who has a surgical placement of a mechanical heart valve. Which of the following statements by the client indicates understanding of the teaching? A. "I will be glad to get back to my exercise routine right away" B. "I will have my prothrombin time checked on a regular bases" C. "I will talk to my dentist about no longer needing antibiotics before dental exams" D. " I will continue to limit my intake of foods containing K+"

A. expected finding for pulmonary edema B. expected finding E. expected finding C. pt would have decreased UO D. pink, frothy sputum is expected

A nurse is completing the admission assessment of a client who has suspected pulmonary edema. Which of the following manifestations are expected findings? (selects all that apply) A. tachypnea B. persistent cough C. increased UO D. Thick, yellow sputum E. orthopnea

A. long term NSAID use can lower platelets B. glucose is not effected by long term platelet use C.although this can be Affected, this value is within normal ranges D.liver is affected by longterm NSAID use, normal value

A nurse is in a clinic is caring for a client who has been on long-term NSAIDs therapy to treat pericarditis. Which of the following lab findings should the nurse report to the HCP? A. platelets 100,000/mm3 B. serum glucose 110 mg/ml C. serum creat 0.7 mg/dl D. amino alanin transferase (ALT) 30 IU/L

C. The Troponin T level will still be evident 10-14 days post MI A. The CK-MB levels are no longer evident after 3 days B.Troponin I levels are no longer present after 7-10 days D. myoglobin levels are no longer present after 24hrs

A nurse on a cardiac unit is reviewing the lab findings of a client who has a diagnosis of MI and reports that his dyspnea began 2 weeks ago. Which cardiac enzyme would confirm an MI 14 days ago? A. CK-MB B.Troponin I C. Troponin T D. Myoglobin

a

A patient diagnosed with acute MI and has nitro IV infusing for relief of pain. The monitoring of which vital sign should receive the highest priority? a. BP b. HR c. temp d. RR

A,C,E

A patient is diagnosed with mitral stenosis and new-onset atrial stenosis and new-onset atrial fibrillation. Which interventions could the nurse delegate to the unlicensed the nurse delegate to the unlicensed assistive personnel (UAP)? Select all assistive personnel (UAP)? Select all that apply. a. obtain and record daily wts b. determine apical and radial pulse rate c. observe for overt signs of bleeding d. teach the patient how to get medic alert device e. obtain and record vital signs

a

After receiving change of shift report about these four client, which client should the nurse assess first? a. the 46 year old with aortic stenosis who takes Digoxin and has new-onset, frequent premature ventricular complexes b. The 55 year old admitted with pulmonary edema admitted with pulmonary edema who recieved furosemide and whose current o2 sat is 94% c. the 68 year old with pericarditis who is reporting sharp, stabbing chest pain when taking deep breaths d. the 79 year old admitted for possible rejection of a heart transplant who has sinus tachycardia, rat 104

2. Diaphoresis (sweating) is a systemic reaction to the MI. The body vasoconstricts to shunt blood from the periphery to the trunk of the body; this, in turn, leads to cold, clammy skin. 1. Midepigastric pain would support a diagnosis of peptic ulcer disease; pyrosis is belching. 3. Intermittent claudication is leg pain secondary to decreased oxygen to the muscle, and pallor is paleness of the skin as a result of decreased blood supply. Neither is an early sign of MI. 4. Jugular vein distension (JVD) and depend- ent edema are signs/symptoms of congestive heart failure, not of MI.

Along with persistent, crushing chest pain, which signs/symptoms would make the nurse suspect that the client is experiencing a myocardial infarction? 1. Midepigastric pain and pyrosis. 2. Diaphoresis and cool clammy skin. 3. Intermittent claudication and pallor. 4. Jugular vein distention and dependent edema.

3. best option is to educate

An 80 y/o pt on the coronary step down unit tells the nurse " I do not need to take that docusate. I never get constipated!" Which action by the nurse is most appropriate? 1. document the med on the client's chart are refused 2. mix the med with food and administer to the client 3. explain that his decreased activity level may cause constipation 4. reinforce that the docusate has been Rx for good reason

D

As one of your clinical assignments, you are assisting an RN with health screening at a health fair. Which individual is at greatest risk for experiencing a stroke? a. A 46-year-old white female with hypertension and oral contraceptive use for 10 years b. A 58-year-old white male salesman who has a total cholesterol level of 285 mg/dl c. A 42-year-old African American female with diabetes mellitus who has smoked for 30 years d. A 62-year-old African American male with hypertension who is 35 pounds overweight

3. visual disturbances may be a sign of digoxin toxicity

During a home visit to an 88 year old pt who is taking digoxin 0.25 mg/day to treat her HF and atrial fibrillation, the nurse obtains this assessment information. Which finding is most imporatnt to communicate to the HCP? 1.apical pulse 68 2.digoxin taken with meals 3.vision that is becoming "fuzzy" 4. lung crackles that clear after coughing

C. the sympathetic nervous system is activated so BP will rise

During an angina attack, the nurse would be most alert for which resulting change in the PT's vital signs? a. decreased HR b.increased temp c. increased BP d. decreased RR

a

Metoprolol (Lopressor) 50 mg bid is ordered for a client who has HTN and chronic renal failure. In teaching the client about medication the nurse should teach teach him to be alert for the development of which possible side effect? a. dizziness b. itching c. tachycardia d. wt loss

3. you want to control pain before you move forward, remember to take vitals before administering sublingual nitro

The ED nurse is caring for a client who was just admitted with L anterior chest pain, possible acute MI. Which action will the nurse take first? 1. insert an IV 2. auscultate heart sounds 3. administer sublingual nitro 4. draw blood for troponin I measurement

1 4 6

The HCP prescribes these actions for the client who was admitted with acute substernal chest pain. Which actions are appropriate to assign to an experienced LPN/LVN who is working in the ED? Select all that apply. 1.attaching cardiac monitor leads 2.giving heparin 5000 units IV push 3. administering morphine sulfate 4mg IV 4. obainting a 12 lead ECG 5. asking the pt for pertinent med HX 6.having the pt chew and swallow aspirin 162 mg

2. the LPN/LVN scope of practice include stat collection

The charge nurse in a long term care facility that employs RN, LPN/LVN, and UAP has developed aplan for the on going assessment of all the residents with a diagnosis of HF. Which activity included in the plan is most appropriate to assign to an LPN/LVN team member? 1. weighing all the residents with HF each morning 2. listening to lung sounds and checking for edema each week 3. reviewing all HF meds with residents each month 4. updating activity plans for residents with HF every quarter

3. A new graduate should be able to complete a pre-procedure checklist and get this client to the catheterization laboratory. 1. This client is at high risk for complications related to necrotic myocardial tissue and will need extensive teaching, so this client should not be assigned to a new graduate. 2. Unstable angina means this client is at risk for life-threatening complications and should not be assigned to a new graduate. 4. Chest pain means this client could be hav- ing a myocardial infarction and should not be assigned to a new graduate.

The charge nurse is making assignments for clients on a cardiac unit. Which client should the charge nurse assign to a new graduate nurse? 1. The 44-year-old client diagnosed with a myocardial infarction. 2. The 65-year-old client admitted with unstable angina. 3. The 75-year-old client scheduled for a cardiac catheterization. 4. The 50-year-old client complaining of chest pain.

1. A pericardiocentesis removes fluid from the pericardial sac and is the emergency treatment for cardiac tamponade. 2. Cardiac enzymes may be slightly elevated because of the inflammatory process, but evaluation of these would not be ordered to treat or evaluate cardiac tamponade. 3. A 12-lead ECG would not help treat the medical emergency of cardiac tamponade. 4. Assessment by the nurse is not collaborative; it is an independent nursing action.

The client diagnosed with pericarditis is experiencing cardiac tamponade. Which collaborative intervention should the nurse anticipate for this client? 1. Prepare for a pericardiocentesis. 2. Request STAT cardiac enzymes. 3. Perform a 12-lead electrocardiogram. 4. Assess the client's heart and lung sounds.

3. If the client bends the leg, it could cause the insertion site to bleed. This is arterial blood and the client could bleed to death very quickly, so this requires immediate intervention. 1. These vital signs are within normal limits and would not require any immediate intervention. 2. The groin dressing should be dry and intact. 4. The nurse must check the neurovascular assessment, and paresthesia would warrant immediate intervention, but no numbness and tingling is a good sign.

The client has just returned from a cardiac catheterization. Which assessment data would warrant immediate intervention from the nurse? 1. The client's BP is 110/70 and pulse is 90. 2. The client's groin dressing is dry and intact. 3. The client refuses to keep the leg straight. 4. The client denies any numbness and tingling.

1. dark and tarry stools indicated GI bleed, AE of both aspirin and clopidogrel

The clinic nurse is evaluating a client who has coronary artery stenting through the right femoral artery a week previously and is taking metroprolol, clopidogrel, and aspirin. Which information reported by the pt is most important to report to the HCP? 1. stools hav been black in color 2.buising is present at the right groin 3. home blood pressure today was 104/52 4. home radial pulse has been 55 to 60

B

The home care nurse visits a patient with chronic heart failure who is taking digoxin (Lanoxin) and furosemide (Lasix). The patient complains of nausea and vomiting. Which action is most appropriate for the nurse to take? a. perform a dipstick urine test for protein b. notify the health care provider immediately c. have the pt eat foods high is K+ d. ask the pt to record a wt every morning

4. The client should take one tablet every five (5) minutes and, if no relief occurs after the third tablet, have someone drive him to the emergency department or call 911. 1. If the tablets are not kept in a dark bottle, they will lose their potency. 2. The tablets should burn or sting when put under the tongue. 3. The client should keep the tablets with him in case of chest pain.

The male client is diagnosed with coronary artery disease (CAD) and is prescribed sublingual nitroglycerin. Which statement indicates the client needs more teaching? 1. "I should keep the tablets in the dark-colored bottle they came in." 2. "If the tablets do not burn under my tongue, they are not effective." 3. "I should keep the bottle with me in my pocket at all times." 4. "If my chest pain is not gone with one tablet, I will go to the ER."

2. symptoms indicate acute hypoxia, so immediate action is needed, other clients should be assessed soon but they are not as acutely ill

The nurse has received a change of shift report about these pt on the coronary step down. Which one will the nurse assess first? 1.26 year old pt with HF caused by congenital mitral stenosis who is scheduled for a balloon valvuloplasty later today 2. 45 year old with constrictive cardiomyopathy who developed acute dyspnea and agitation about 1 hour before shift change 3 56 year old who underwent coronary angioplasty and stent placement yesterday and has reported occasional chest pain since the procedure 4.77 year old transferred from the ICU 2 days ago after coronary art bypass grafting and has a temp of 100.6

4. Scheduling activities and rest periods allows the client to participate in his or her own care and addresses the desired outcome. 1. Measuring the intake and output is an appropriate intervention to implement for a client with CHF, but it does not address getting the client to tolerate activity. 2. Dietary sodium is restricted in clients with CHF, but this is an intervention for de- creasing fluid volume, not for increasing tolerance for activity. 3. Daily weighing monitors fluid volume status, not activity tolerance.

The nurse has written an outcome goal "demonstrates tolerance for increased activity" for a client diagnosed with congestive heart failure. Which intervention should the nurse implement to assist the client to achieve this outcome? 1. Measure intake and output. 2. Provide two (2)-g sodium diet. 3. Weigh client daily. 4. Plan for frequent rest periods.

2. lightheadedness and syncope may indicate that the pt's hr is either too fast or too slow, affecting brain perfusion and causing risk for complications such as falls

The nurse in the cardiovascular clinic receives calls from four pts. Which pt should be scheduled to be seen most urgently? 1. pt with peripheral arterial disease who complains of leg cramps when walking 2. pt with atrial fibrillation who reports episodes of lightheadedness and syncope 3. pt with new permanent pacemaker who has severe itchiness at the wound site 4. pt with angina who took nitro twice in the last week while exercising

2, 4, 3, 1

The nurse is ambulating a cardiac surgery client whose heart rate suddenly increases to 146. in which order will the nurse take the following actions? 1.call the client's HCP 2.have the client sit down 3.check the pt BP 4. administer oxygen

C

The nurse is assessing the client with a cardiac issue. Which symptoms support the diagnosisi of infective endocarditis instead of pericarditis or Rheumatic carditis? a. friction rub auscultated at the left lower sternal border b. pain aggravated by breathing, coughing, and swallowing c. splinter hemorrhages d. thickening of endocardium

D

What subjective data from a patient would help distinguish a hemorrhagic stroke from a thrombotic stroke? a. reports presence of motor weakness b. family history of hypertension c. uses oral contraceptives d. sudden onset of severe headache

B

When teaching a pt with CAD about risk factors, which would the nurse include regarding cholesterol? a. cholesterol causes an increased in serum HDL b. cholesterol is found in both plant and animal sources c. cholesterol should be eliminated because it causes the disease process d. cholesterol decreases when saturated fats are replaced with unsaturation fats

3. Intermittent claudication is a sign of generalized atherosclerosis and is a marker of atherosclerosis. 1. A change in bowel movements may indicate cancer but not atherosclerosis. 2. A headache is not a sign/symptom of atherosclerosis. 4. Atherosclerosis indicates arterial involvement, not venous involvement.

Which assessment data would cause the nurse to suspect the client has atherosclerosis? 1. Change in bowel movements. 2. Complaints of a headache. 3. Intermittent claudication. 4. Venous stasis ulcers.

B

Which clinical indicator does the nurse ID when assessing a pt with hemiplegia? a. paralysis of both lower extremities b. paralysis of one side of the body c. paralysis of both upper extremities d. paralysis of the upper and lower extremities

2. the cardiac nurses would be familiar with the other meds and interventions but the float would be best with furosemide because it is a well known drug

Which pt is best for the coronary care charge nurse to assign to a float RN who has come for the day from the general med-surg unit? 1.Pt requiring discharge teaching about coronary art stenting before going home today. 2. Pt receiving IV furosemide to treat acute L vent failure 3. pt who just transferred in from the radiology department after coronary angioplasty 4.pt just admitted with unstable angina who has orders for heparin infusion and aspirin

D

Which should the nurse assess to determine the effectiveness of a vasodilator administered to a PT? a. pulse b. breath sounds c. cardiac output d. BP

1 3 5 6

Which topics will the nurse plan to include in discharge teaching for a pt who has been admitted with HF? Select all that apply. 1.how to monitor and record daily wt 2.importance of stopping exercise if HR increases 3.symptoms of worsening HF 4.purpose of chronic antibiotic therapy 5.how to read food labels for Na content 6. date and time for follow up appointments

B

While collecting a patient history, the nurse identifies which of the following risk factors as an increased risk for development of infective endocarditis? a. alcoholism b. IV drug abuse c. Glamerulonephritis d. atherosclerosis

4. assisting in hygiene is part of the UAPs job role

a resident in a long term care facility who has venous stasis ulcers is treated with Unna boot.. Which nursing activity included in the resident's care is best for the nurse to delegate to the UAP? 1.teaching family members signs of infection 2.monitoring capillary perfusion once every 8 h 3. evaluating foot sensation and movement each shift 4. assisting the pt in cleaning around the boot

1.since they are going to be going down the pt throat, the pt needs to be NPO for the next 12 hours

at 1000, a hospitalized pt receives a new order for transesophageal echocardiography as soon as possible. Which action will the nurse take first? 1.put the pt on NPO status 2.teach the pt about the procedure 3.insert an IV 4.attach the pt to a cardiac monitor

4. because chest pain lasting more than 12 hours usually indicated irreversible myocardial necrosis, fibrinolytic drugs are not recommended

At 2100, the nurse admits a 63 y/o pt with a Dx of acute MI. Which finding is most important to communicate to the HCP who is considering the use of fibrinolytic therapy with tissue plasminogen activator (alteplase) for the client? 1. the pt was treated with alteplase about 8mon ago 2. the pt takes famotidine for gastroesophageal reflux disease 3.the pt has ST seg elevations on the ECG 4.the pt reports having continuous pain since 0800

c

At the end of the visit, the provider prescribes hydrochlorothiazide (HydroDIURIL) 25 mg Po each morning to manage HTN. Which statement do you include when teaching the patient about this drug? a. "This is a loop diuretic that decreases sodium reabsorption" b. "HydroDIURIL is a potassium sparing diuretic that helps prevent the loss of essential potassium" c. "You may need to consume foods rich in potassium, such as bananas and orange juice" d. "a potassium supplement will be prescribed along with this drug"

A. Ace inhibitor D. diuretic E. Digoxin

During the evening shift, the pt has a bedside echocardiogram which reveals an ejection fraction of 30%. Base on this finding, which med might the provider order? (select all the apply) a. Lisinopril (Zestril) 5mg PO daily b. Zocor (simvastatin) 40mg PO daily in the evening c. Lopressor 50 mg PO daily d. Furosemide (Lasix) 20 mg IV push daily e. Digoxin (Lanoxin) 0.25 mg PO daily

1. elevated BP puts stress of graft suture and could lead to rupture

During the initial post op assessment of the pt who has just been trandferred to the PACU after repair of an abdominal aortic aneurysm, the nurse obtains these data. Which finding has the most immediate implications for the pt's care? 1.arterial line implicates a BP of 190/122 2. frequent premature atrial contractions 3. there is no response to verbal stimulation 4. UO is 40 ml of amber urine

c

A 70 year old pt with a hx of L vent failure arrives for a scheduled clinic appointment and tells the nurse "My feet are killing me, These shoes got so tight" The nurse's best initial action would be to: a. weigh the pt b. notify the Dr c. listen to he pt's breath sounds d. count the pulse

4. the goal of pain management is to eliminate pain, pain of a 1 is still pain

A nurse has given morphine sulfate 4mg IV to a pt who is having an acutre MI. When evaluating the pt's response 5 min after giving the med, which finding indicates a need for immediate further action? 1.BP decreases from 114/65 to 106/58 2. RR decreased from 18 to 12 3. cardiac monitor indicating sinus rhythm at rate of 96 beats/min 4. persisting chest pain at level of 1

B. child with streptococcal pharyngitis is at risk for rheumatic fever which can result in rheumatic endocarditis A. not at risk unless they develop a rheumatic fever C.not at risk unless they get a rheumatic fever D.at increased risk fo infective endocarditis, only at risk if they get a rheumatic fever

A nurse ic caring for four pt. Which of the following pts should the nurse ID as being at risk of developing rheumatic endocarditis? A. older adults who has chronic obstructive pulmonary disease B. child who has streptococcal pharyngitis C. middle ages adult who has lupus erythematosus D. young adult who recently received a body tattoo

A. stable angina can be relived by rest and nitro B.pain with an MI last longer than 30 minutes and an opioid is used to relieve pain c. there is no specific activity that can cause an MI d. the pain of stable angina usually occurs in 15 minutes or less

A nurse is admitting a client who has a suspected MI and hx of angina. Which of the following findings will help the RN distinguish stable angina from MI? A. stable angina can be relieved with rest and ntroglycerin B. the pain of an MI resolves in less than 15 min C. the type of activity that causes an MI can be ID D. stable angina can occur for longer than 30 min

A. Apririn decreases platelet aggregtion that can cause an MI B. one aspirin per day will not relieve ischemic pain C. aspirin does not resolve clots D. other meds can cause head aches, but one aspirin a day is not an analgesic

A nurse is caring for a client who asks why they were prescribed a daily aspirin. What response should the nurse make? A. "Aspirin reduces the formation of blood clots that could cause a heart attack" B. "Aspirin relieves pain due to ischemia" C. "Aspirin dissolves clots that are forming in for coronary arts" D. Aspirin relieves headaches that are caused by other meds"

D.Friction rub heard with pericarditis A.petechiae is expected for endocarditis B. murmur is expected with myocarditis and endocarditis C.rash is expected with rheumatic endocarditis

A nurse is caring for a client who has pericarditis. Which of the following findings should the nurse expect? A.petechiae B.murmur C.rash D.friction rub

C.The greatest risk is injury due to very high blood pressure, that must be controlled before further action is taken A.not priority B.not priority D. not priority

A nurse is caring for a client who si admitted to the ED with a BP of 266/147. Pt reports headache and double vision. Pt states, " I ran out of my diltiazem 3 days ago, and I am unable to purchase more." Which of the following actions should the nurse take first? A. administer acetaminophen for H/A B. provide teaching about not abruptly stopping and antihypertensive C.obtain IV access and prepare to administer an IV antihypertensive D. call social services for a referral for financial assistance

C. neck vein distention is an expected finding with mitral valve insufficiency A. S3 would be heard, S4 would be expected with atrial stenosis B. petechiae is an expected finding for a patient with infective endocarditis D. hepatomegaly not splenomegaly would be expected with leftsided heart failure

A nurse is completing the admission physical assessment of a client who has a mitral valve insufficiency. Which of the following findings should the nurse expect? A. S4 heart sounds B. petechiae C. neck vein distension D. splenomegaly

C. According to ABC smoking would be first A. this isimportant but not the top priority B. this is good but not top priority D. this is good but not top

A nurse is presenting a community education program on recommended lifestyle changes to prevent angina and MI. Which of the following changes should the nurse recommend by made first? A. diet modification B. relaxation C. smoking cessation D. taking omega-3 capsules

A. provides a visual guide and help the patient plan out the day B. glasses and cans can vary is size C. vague D. does not assist in accurate measurement

A nurse is providing discharge teaching for a client who has HF and is on a fluid restriction of 2,000ml/day. The client asks how to determine the amount of fluid they allow. Which statement is an appropriate response by the nurse? A. "pour the amount of liquid you drink into an empty 2L bottle and keep track of how much you drink" B. "each glasss contains 8oz. There are 30ml in an oz, so you can have 8 glasses or cups per day" C. "this is the same as two quarts, or two pots of coffee" D. "take sips of water or ice chips so you will not take too much fluid

A. to prevent nocturia B. may interrupt sleep if taken at this time C.may interrupt sleep if taken at this time D. may interrupt sleep if taken at this time

A nurse is providing discharge teaching for a client who has a prescription for furosemide 40mg PO daily. The nurse should instruct the client to take this med at which of the following times of day? A.morning B.immediately after lunch C.immediately before dinner D.bedtime

B. teach pt to monitor their HR and tell provider A. K+ sparing diuretic so the pt can because hyperkalemic is they eat too much K+ C.substitutes are usually high in K+ so that can lead to hyperkalemia due to using a K+ sparing diuretic D. must keep taking med

A nurse is providing teaching for a client who has a new diagnosis of HTN and a new Rx for spironolactone 25 mg/day. Which of the following statements by the client indicates an understanding of the teaching? A. "I should eat a lot of fruits and veggies especially bananas and potatoes" B. "I will report any changes in heart rate to my provide" C. "I should replace the salt shaker on my table with a salt subsistute"" D. "I will decrease the dose of this med when I no longer have H/A and facial redness"

4. LDL is elevated if it stays that way then intervention will have to take place

A nurse is reviewing lab results for a pt with and elevated cholesterol who is taking atorvastatin. Which result is most important to discuss with the HCP? 1.K+ is 3.4 2.BUN is 9 3.AST is 30 4. LDL cholesterol is 170

A. hx of malformations is a risk C. HTN is a risk E, murmur indicates turbulent blood flow, usually a due to valve disease B. not a risk factor D. a sudden wt gain would indicate left sided failure

A nurse is reviewing the health record of a client who is being evaluated for possible valvular heart disease. The nurse should recognize which of the following data as risk factors for this condition? (select all that apply) A. surgical repair of an atrial septal defect at age 2 B.measles infections during childhood C. HTN for 5 years D. Wt gain of 10lbs in one year E. diastolic murmur present

B. movie theater popcorn is high is Na D. drinking more than 24oz per day for a male is a risk A.nonfat is good C. exercise is good E. relaxtation

A nurse is screening for hypertension. The nurse should ID that which of the following actions by the client will increase their risk for HTN? A. drinking 8oz glass of nonfat milk daily B eating popcorn at the movie theater C. walking one mile a day at 12 minute pace D. consuming 36oz of beer daily E.getting a massage once a week

C. VADs are used to prolong the life of HF pt who do not respond to meds A. VADs are usually given to pt waiting for transplants B. VADs are pumps to get blood around the body D. VADs are contraindicated for pt with chronic lung disease

A nurse is talking with a pt who has class I HF and asks about obtaining a ventricular assist device (VAD). Which of the following statements should the nurse make? A. "VADs are only implanted during a heart transplant" B. "A VAD helps to pace the heart" C. "VADs are used when HF is not responsive to meds" D. A VAD is useful for pt who also have chronic lung disease"

A. helps to keep track of fluid loss of gain E. prevents fluid retention B. would want to increase K+ C. muscle weakness of Dig is a sign of toxicity D. hold is <60

A nurse is teaching a client who has HF and new Rx for furosemide and digoxin. Which of the following information should the nurse include? (Select all theat apply) A.weigh daily, first thing each morning B. decrease intake of K+ C. expect muscle weakness while taking digoxin D. hold digoxin if HR <70 E. decrease Na intake

D. The client is advised to notify the provider if bradycardia occurs A. administered orally not SQ B. does not affect bleeding or clotting time C.ringing in the ears is not an adverse reaction to this med (dryness is)

A nurse is teaching a clinet who has angina about metoprolol. Which of the following indicates the patient understood the teachings? A. "I should place this tab under my tongue" B. "I shoul have my clotting time checked weekly" C. "I will report any ringing in my ears" D. "I will call my doctor if my pulse is less than 60"

a. make sure patient can swallow

A patient has dysphagia following a stroke. Before allowing him to eat, which action should the nurse take first? a.Check the patient's gag reflex. b.Request a soft diet with no liquids. c.Place the patient in high-Fowler's position. d.Test the patient's ability to swallow with a small amount of water.

B, C, D, E

A patient is diagnosed with prehypertension. He asks you how this could happen since he feels fine. Which points do yo include in your teaching plan? (select all that apply) a. checking BP only at the clinic to ensure accuracy b. making lifestyle changes to control BP c. exercising and WT loss to decrease the need for BP meds d. limiting smoking and caffeine to moderate use e. alternate therapies such as relaxation techniques to help decrease stress associated with HTN

C

A patient recieving Digoxin and Lasix PO daily. Based on these meds, which instruction is the highest priority for the nurse to provide to the pt? a. be sure to increase your intake of Ca containing foods each day b. avoid green leafy veggies c. drink orange juice and eat bananas or melons each day d. avoid foods high in sodium

4. because use of metformin may lead to acute lactic acidosis when clients undergo procedures that use iodine-based contrast dye, metformin should be held for 24 hours before and 48 hours after

A patient who is scheduled for coronary arteriogram is admitted to the hospital on the day of the procedure. Which client info is most important for the nurse to communicate to the hCP before the procedure? 1.glucose 144 2. cardiac monitor shows sinus bradycardia, 56 3. client reports chest pain that occurred yesterday 4. client took metformin 500 mg this morning

B

A patient with a history of chronic heart failure is hospitalized with severe dyspnea and a dry, hacking cough. Assessment findings include pitting edema in both ankles, BP 170/100 mm Hg, pulse 92 beats/minute, and respirations 28 breaths/minute. In speaking with the interprofessional team, which explanation, if made by the nurse, is most accurate? a. "The assessment indicates that venous return to the heart is impaired, causing a decrease in cardiac output." b. "The manifestations indicate impaired emptying of both the right and left ventricles, with decreased forward blood flow." c. "The myocardium is not receiving enough blood supply through the coronary arteries to meet its oxygen demand." d. "The patient's right side of the heart is failing to pump enough blood to the lungs to provide systemic oxygenation."

a

A patient with cardiovascular disease is prescribed a potassium wasting diuretic. Which food should the patient consume to help prevent hypokalemia? a. baked potatoes b. raw avacado c. dried figs d. red apples

B

A patient with left-sided heart failure is prescribed oxygen at 4 L/min per nasal cannula, furosemide (Lasix), spironolactone (Aldactone), and enalapril (Vasotec). Which assessment should the nurse complete to best evaluate the patient's response to these drugs? a. measure UO b. auscultate lung sounds c. measure BP d. calculate fluid balance

B

A physician orders Lopressor 12.5 mg PO daily. The drug is only supplied in 25 mg tablets. What instruction is essential for the nurse to provide to the patient? a. Take one tablet every other day b. use a pill splitter to cut the tablet in half and take 1/2 tab eachday c. ask the physician to change the med d. take this med at night when going to bed

2, 3, 4, 6, 7, 8

A pt admitted with Digoxin toxicity. Select all of the signs and symptoms below which are associated with digoxin toxicity. 1. seizures 2. bradycardia 3. vomiting 4. diarrhea 5. tachyacrdia 6. yellow halos in the visual fields 7. blurred vision 8. headache

A ace cough

A pt is being treated for HTN reports having a persistent dry hacking cough. The nurse IDs this is side effect of which med? a. ACE inhibitor b. thiazide diuretics c. Ca channel blockers d. Angiotensin receptor blockers

a. therapeutic level 0.5-2

A pt ordered digoxin 1.25 mg daily at 9am. The digoxin level this morning is 1.8ng/ml. Through his understanding of this med, the nurse is aware that: a. level is therapeutic b. the level is elevated c. the level is subtherapuetic d. digoxin should not be evaluated by therapuetic levels

2. B-type natriuretic peptide levels increase in pts with poor left vent function and symptomatic HF and can be used to differentiate HF from other causes of dyspnea and fatigue such as pneumonia

A pt seen in the clinic with SOB and fatigue is being evaluated for possible diagnosis of HF. Which lab result will be most useful to monitor? 1. serum K+ 2. B type natriuretic peptide 3. BUN 4. Hematocrit

A, C, D (of stomach gets too full they can aperate)

A pt who has had a CVA receiving continous nasogastric tube feedings, Which of the following should the nurse implement to prevent aspiration pneumonia. Select all that apply. a. HOB up at least 45 degrees at all times b. separate from infeccted patients c. verify tube placement at the beginning of each shift d. check residual volume every 4 hrs e. offer bedpan q 4 h f. allow the client to lie on the left side periodically

2. troponin levels are elevated 3 hours after the onset of MI and are very specific to cardiac muscle injury

A pt who has just arrived in the ED reports substernal and left arm discomfort that has been going on for about 3h. Which lab test will be most useful in determining whether the nurse should anticipate implementing the acute coronary syndrome standard protocol? 1. CK-MB 2. Troponin I 3. myoglobin 4.c-reactive protein

3. home blood pressures are recommended by the American Heart Association for people with HTN

A pt whose systolic BP is always higher than 140 in the clinic tells the nurse "my BP at home is always fine!" what action should the nurse take next? 1.instruct the pt about the effects of untreated high BP on the cardiovascular and cerebrovascular system 2. educate the pt about lifestyle changes such as low sodium diet, exercise, and restricting alcohol 3. ask the pt to obtain BPs twice daily with an automatic cuff at home and bring the result in 4. provide th pt with a hand out describing the various types of meds for HTN

B

A pt with CHF is ordered 100 ml fluid resriction/ 24hr and 1 gm sodium restriction diet. Which statement by the pt indicateds good understanding of these resrictions? a. "I should avoid all milk products" b. I guess I can't eat canned soup anymore" c. "At least I can as much bread as i like" d. I need to watch my fresh veggies because I know they have a lot of sodium"

3. the pt history and symptoms indicate that acute arterial occlusion has occurred. because its important to return blood flow rapidly the HCP has to be notified

A pt with endocarditis with vegetation on the mitral valve suddenly reports severe left foot pain. The nurse notes that no pulse is palpable in the left foot and that it is cold and pale. Which action should the nurse take next? 1.lower the pt l foot below heart level 2. administer oxyegn 4L/min 3. notify the HCP about change in status4. reassure the pt that embolization is common in endocarditis

3. chronic constipation is a common AE of ranolazine

A pt with stable angina has a Rx for ranolazine 500mg BID. Which pt finding is most important for the nurse to discuss with the HCP? 1.HR 52 2.pt is also taking carvediolol for angina 3.pt reports having chronic constipation 4.BP 106/56

3. the other choices are common side effects are starting the med but bradycardia indicated a need for a decrease in dose

Two weeks ago, a pt with HF received a new Rx for carvedilol 12.5 mg PO. Which finding by the nurse is of most concern? 1.reports of increased fatigue and activity intolerance 2.wt increase of 0.5kg over 1 week 3. sinus bradycardia at rate of 48 4. traces of edema noted over both ankles

B

What is the greatest risk for a patient with dysfunction of cranial nerves IX and X? a. dehydration b. aspiration pneumonia c. constipation d. Wt loss

2, 3, 4

What should the nurse include in teaching for the client being discharged on furosemide (Lasix)? Select all that apply. 1. med may be taken any time during the day 2. increase intake of potatoes, oranges, strawberries, tomatoes, and avocados 3. check BP daily. call if is lower than normal 4. make position changes slowly 5. take this med 1/2 hour before meals

A. you make the most cholesterol at night B. grapefruit effects the drug D. muscle injury

What should the nurse include in the teaching for the patient being discharged on simvastatin (Zocor)? select all that apply. a. administer once daily in the evening b. avoid grapefruit juice and grapefruit c. hard candy can help alleviate side effect of dry throat and cough d. report unexplained muscle pain, weakness and tenderness e. this med is safe with pregnancy

4. chest pain in a pt undergoing a stress test indicated myocardial ischemia and means you need to stop to avoid ongoing ishcemia

When the nurse is monitoring a 53 y/o pt who is undergoing a treadmill stress test, which finding will require the most immediate action? 1.BP 152/88 2.HR 134 3.o2 91% 4.chest pain level 3

3. Uncontrolled hypertension is a risk factor for hemorrhagic stroke, which is a ruptured blood vessel inside the cranium. 1. This glucose level is elevated and could predispose the client to ischemic neurologi- cal changes due to blood viscosity, but it is not a risk factor for a hemorrhagic stroke. 2. A carotid bruit predisposes the client to an embolic or ischemic stroke but not to a hemorrhagic stroke. 4. Cancer is not a precursor to developing a hemorrhagic stroke

Which assessment data would indicate to the nurse that the client would be at risk for a hemorrhagic stroke? 1. A blood glucose level of 480 mg/dL. 2. A right-sided carotid bruit. 3. A blood pressure of 220/120 mm Hg. 4. The presence of bronchogenic carcinoma.

2. The murmur associated with mitral valve insufficiency is loud, high- pitched, rumbling, and holosystolic (occurring throughout systole) and is heard best at the cardiac apex. 1. This would be expected with mitral valve stenosis. 3. This would be expected with mitral valve prolapse. 4. This would be expected with aortic regurgitation.

Which assessment data would the nurse expect to auscultate in the client diagnosed with mitral valve insufficiency? 1. A loud S1, S2 split, and a mitral opening snap. 2. A holosystolic murmur heard best at the cardiac apex. 3. A midsystolic ejection click or murmur heard at the base. 4. A high-pitched sound heard at the third left intercostal space.

3. Troponin is the enzyme that elevates within 1 to 2 hours. 1. CPK-MB elevates in 12 to 24 hours. 2. LDH elevates in 24 to 36 hours. 4. WBCs elevate as a result of necrotic tissue, but this is not a cardiac enzyme.

Which cardiac enzyme would the nurse expect to elevate first in a client diagnosed with a myocardial infarction? 1. Creatine kinase (CK-MB). 2. Lactate dehydrogenase (LDH). 3. Troponin. 4. White blood cells (WBCs).

1. A low-fat, low-cholesterol diet will help decrease the buildup of atherosclerosis in the arteries. 2. Walking will help increase collateral circulation. 4. Stress reduction is encouraged for clients with CAD because this helps prevent excess stress on the heart muscle. 5. Increasing fiber in the diet will help remove cholesterol via the gastrointestinal system. 3. salt should be restricted in the diet of a client with hypertension, not CAD

Which client teaching should the nurse implement for the client diagnosed with coronary artery disease? Select all that apply. 1. Encourage a low-fat, low-cholesterol diet. 2. Instruct client to walk 30 minutes a day. 3. Decrease the salt intake to two (2) g a day. 4. Refer to counselor for stress reduction techniques. 5. Teach the client to increase fiber in the diet.

1. African Americans have twice the rate of CVAs as Caucasians and men have a higher incidence than women; African Americans suffer more extensive damage from a CVA than do people of other cultural groups. 2.Females are less likely to have a CVA than males, but advanced age does increase the risk for CVA. The Oriental population has a lower risk, possibly as a result of their relatively high intake of omega-3 fatty acids, antioxidants found in fish. 3.Caucasians have a lower risk of CVA than do African Americans, Hispanics, and Native Pacific Islanders. 4.Pregnancy is a minimal risk for having a CVA.

Which client would the nurse identify as being most at risk for experiencing a CVA? 1. A 55-year-old African American male. 2. An 84-year-old Japanese female. 3. A 67-year-old Caucasian male. 4. A 39-year-old pregnant female.

2. Clients with Marfan's syndrome have life-threatening cardiovascular problems, including mitral valve prolapse, progressive dilation of the aortic valve ring, and weakness of the arterial walls, and they usually do not live past the age of 40 because of dissection and rupture of the aorta. 1. Congestive heart failure does not predispose the female client to having a mitral valve prolapse. 3. Atrial fibrillation does not predispose a client to mitral valve prolapse. 4. A client with Down syndrome may have congenital heart anomalies but not mitral valve prolapse.

Which client would the nurse suspect of having a mitral valve prolapse? 1. A 60-year-old female with congestive heart failure. 2. A 23-year-old male with Marfan's syndrome. 3. An 80-year-old male with atrial fibrillation. 4. A 33-year-old female with Down syndrome.

1. because endocarditis is a concern with valvular disease, an elevated temp indicated a need for further assessment

Which finding in a client with aortic stenosis will be most important for the nurse to report to the HCP? 1. temp of 102.1 2.loud systolic murmur over sternum 3.BP of 110/88 4. weak radial and pedal pulses

2. Loop diuretics cause potassium to be lost in the urine output. Therefore, the nurse should assess the client's potassium level, and if the client is hypokalemic, the nurse should question administering this medication. 1. The nurse should always assess the apical (not radial) pulse, but the pulse is not affected by a loop diuretic. 3. The glucometer provides a glucose level, which is not affected by a loop diuretic. 4. The pulse oximeter reading evaluates peripheral oxygenation and is not affected by a loop diuretic.

Which intervention should the nurse implement when administering a loop diuretic to a client diagnosed with coronary artery disease? 1. Assess the client's radial pulse. 2. Assess the client's serum potassium level. 3. Assess the client's glucometer reading. 4. Assess the client's pulse oximeter reading.

1. Adherence to lifestyle modifications is enhanced when the client receives support from significant others. 2. Tobacco use is the most significant modifiable risk factor that contributes to the development of atherosclerosis. 3. A sedentary lifestyle should be discouraged; daily walking or swimming is encouraged. 4. This is an unrealistic intervention. The nurse needs to help the client learn ways to deal with stressful situations, not avoid the situations. 5. Isometric exercises are weight-lifting exercises, which should be discouraged; isotonic exercises, such as walking or swimming, are encouraged.

Which interventions should the nurse discuss with the client diagnosed with atherosclerosis? Select all that apply. 1. Include the significant other in the discussion. 2. Stop smoking or using any type of tobacco products. 3. Maintain a sedentary lifestyle as much as possible. 4. Avoid stressful situations. 5. Daily isometric exercises are important.

4. Activity intolerance is priority for the client with myocarditis, an inflammation of the heart muscle. Nursing care is aimed at decreasing myocardial work and maintaining cardiac output. 1. Anxiety is a psychosocial nursing diagnosis, which is not a priority over a physiological nursing diagnosis. 2. Antibiotic therapy does not result in injury to the client. 3. Myocarditis does not result in valve damage (endocarditis does), and there would be decreased, not increased, cardiac output.

Which nursing diagnosis would be priority for the client diagnosed with myocarditis? 1. Anxiety related to possible long-term complications. 2. High risk for injury related to antibiotic therapy. 3. Increased cardiac output related to valve regurgitation. 4. Activity intolerance related to impaired cardiac muscle function.

B

Which patient is at greatest risk for having a stroke? A.An obese 45-year-old Native American B.A 65-year-old black man with hypertension C.A 35-year-old Asian American woman who smokes D.A 32-year-old white woman taking oral contraceptives

2. Bacteria enter the bloodstream from invasive procedures, and sterile platelet-fibrin vegetation forms on heart valves. The mitral valve is on the left side of the heart and, if the vegetation breaks off, it will go through the left ventricle into the systemic circulation and may lodge in the brain, kidneys, or peripheral tissues. 1. Pulmonary embolus would occur with an embolization of vegetative lesions from the tricuspid valve on the right side of the heart. 3. Coughing up blood (hemoptysis) occurs when the vegetation breaks off the tricuspid valve in the right side of the heart and enters the pulmonary artery. 4. Deep vein thrombosis is a complication of immobility, not of a vegetative embolus from the left side of the heart.

Which potential complication should the nurse assess for in the client with infective endocarditis who has embolization of vegetative lesions from the mitral valve? 1. Pulmonary embolus. 2. Cerebrovascular accident. 3. Hemoptysis. 4. Deep vein thrombosis.

1. Paroxysmal nocturnal dyspnea is a sudden attack of respiratory distress, usually occurring at night because of the reclining position, and occurs in valvular disorders. 2. This is an abnormal condition in which a client must sit or stand to breathe comfortably and occurs in valvular disorders. 3. Coughing occurs when the client with long-term valvular disease has difficulty breathing when walking or performing any type of activity. 4. Pericardial friction rub is a sound auscultated in clients with pericarditis, not valvular heart disease. 5. Pulsus paradoxus is a marked decrease in amplitude during inspiration. It is a sign of cardiac tamponade, not valvular heart disease.

Which signs/symptoms should the nurse assess in any client who has a long-term valvular heart disease? Select all that apply. 1. Paroxysmal nocturnal dyspnea. 2. Orthopnea. 3. Cough. 4. Pericardial friction rub. 5. Pulsus paradoxus.

2. The American Heart Association recommends a low-fat, low-cholesterol diet for a client with coronary artery disease. The client should avoid any fried foods, especially meats, and bake, broil, or grill any meat. 1. According to the American Heart Association, the client should not eat more than three (3) eggs a week, especially the egg yolk. 3. The client should drink low-fat milk, not whole milk. 4. Pork products (bacon, sausage, ham) are high in sodium, which is prohibited in a low-salt diet, not a low-cholesterol, low- fat diet.

Which statement by the client diagnosed with coronary artery disease indicates that the client understands the discharge teaching concerning diet? 1. "I will not eat more than six (6) eggs a week." 2. "I should bake or grill any meats I eat." 3. "I will drink eight (8) ounces of whole milk a day." 4. "I should not eat any type of pork products."

a

While performing blood pressure screening at a health fair, the screening at a health fair, the nurse counsels which person as nurse counsels which person as having the greatest risk for having the greatest risk for developing hypertension? a. a 56 year old man whose father died at age 62 from a stroke b.a 30 year old female with a stressful job as advertising agent, lives alone c. a 68 year old man who uses herbal remedies to treat his enlarged prostate gland d. a 43 year old man who travles extensively with his job and exercises only on weekends

2.The new RNs education and orientation would have included safe administration of IV meds. Other pt require more complex assessment of teaching by an RN with experience

While working on the cardiac step down unti, the nurse is precepting an newly grad RN who has been in a 6week orientation program. Which pt will be best to assign to the new grad? 1. 19year old pt with rheumatic fever who needs discharged teaching before going home 2.33 year old pt admitted a week ago with endocarditis who will be receiving a scheduled dose of ceftriaxone 2 g IV 3. 50 yea old pt with newly diagnosed stable angina who has many questions 4.75 year old pt who has just been tranferred to the unit. after undergoing coronary artery bypass grafting

2. taking lisinopril with sacubitril-valartan will cause angioedema so they are contraindicated

the nurse is caring for a pt who has HF and has a new Rx for sacubitril-valartan. Which pt information is most important to discuss with the HCP before administration of the med? 1.o2 is 92% 2. pt takes lisinopril 10 mg/day 3. bp 150/90 4. K+ is 3.3

D

In assessing a 65 year old pt admitted for heart failure, the nurse IDs which of the following risk factors as unmodifiable? a. stress b. smoking c. uncontrolled diabetes d. family hx of CAD

B

In managing the care of a patient with acute pericarditis, the nurse ensures monitoring with which diagnostic procedure occurs daily? a. chest x-ray b. ECG c. CT scan d. blood culture

C

In planning care for a pt having an attack of unstable angina, the nurse would plan to administer which of the following meds? a. Lovenox b. Furosemide c. Heparin and Nitro D. Propranolol and diltiazem

4. The nurse must complete a neurological assessment to help determine the cause of the headache before taking any further action. 1. The nurse should not administer any medication to a client without first assessing the cause of the client's complaint or problem. 2. An MRI scan may be needed, but the nurse must determine the client's neurological status prior to diagnostic tests. 3. Starting an IV infusion is appropriate, but it is not the action the nurse should implement when assessing pain, and 100 mL/hr might be too high a rate for an 85-year-old client.

The 85-year-old client diagnosed with a stroke is complaining of a severe headache. Which intervention should the nurse implement first? 1. Administer a nonnarcotic analgesic. 2. Prepare for STAT magnetic resonance imaging (MRI). 3. Start an intravenous infusion with D5W at 100 mL/hr. 4. Complete a neurological assessment.

2. Statin medications should be taken in the evening for best results because the enzyme that destroys cholesterol works best in the evening and the medication enhances this process. 1. A statin medication can be taken with food or on an empty stomach 3. Cholesterol-reducing medications can cause serious liver problems, and if a client has muscle pain, it is an adverse effect that should be reported to the HCP. 4. The cholesterol level is checked every few months, not on a daily basis.

The HCP prescribes an HMG-CoA reductase inhibitor (statin) medication to a client with CAD. Which should the nurse teach the client about this medication? 1. Take this medication on an empty stomach. 2. This medication should be taken in the evening. 3. Do not be concerned if muscle pain occurs. 4. Check your cholesterol level daily.

2. NSAIDs other than aspirin inhibit the beneficial effect of aspirin in coronary artery disease

The HCP telephones the nurse with a new Rx for a pt with angina who is already taking aspirin. Which med is most important to clarify further with the HCP? 1. Clopidogrel 75mg/day 2. Ibuprofen 200mg q4 PRN 3. Metoprolol 50 mg/day 4. Nitroglycerin patch 0.4mg/hr

3. This client is exhibiting signs/symptoms of shock, which makes this client the most unstable. An experienced nurse should care for this client. 1. This client is stable because discharge is scheduled for the following day. There- fore, this client does not need to be as- signed to the most experienced registered nurse. 2. This client is more in need of custodial nursing care than care from the most experienced registered nurse. Therefore, the charge nurse could assign a less experienced nurse to this client. 4. These complaints usually indicate muscular or pleuritic chest pain; cardiac chest pain does not fluctuate with inspiration.

The charge nurse is making shift assignments for the medical floor. Which client should be assigned to the most experienced registered nurse? 1. The client diagnosed with congestive heart failure who is being discharged in the morning. 2. The client who is having frequent incontinent liquid bowel movements and vomiting. 3. The client with an apical pulse rate of 116, a respiratory rate of 26, and a blood pressure of 94/62. 4. The client who is complaining of chest pain with inspiration and a nonproductive cough.

2. Because this client is being discharged, it would be an appropriate assignment for the new graduate. 1. This client requires teaching and an understanding of the pre-procedure interventions for diagnostic tests; therefore, a more experienced nurse should be assigned to this client. 3. Supraventricular tachycardia (SVT) is not life threatening, but the client requires intravenous medication and close monitor- ing and therefore should be assigned to a more experienced nurse. 4. A client with atrial fibrillation is usually taking the anticoagulant warfarin (Coumadin), and the therapeutic INR is 2 to 3. An INR of 5 is high and the client is at risk for bleeding.

The charge nurse is making shift assignments. Which client would be most appropriate for the charge nurse to assign to a new graduate who just completed orientation to the medical floor? 1. The client admitted for diagnostic tests to rule out valvular heart disease. 2. The client three (3) days post-myocardial infarction being discharged tomorrow. 3. The client exhibiting supraventricular tachycardia (SVT) on telemetry. 4. The client diagnosed with atrial fibrillation who has an INR of five (5).

4. This response explains in plain terms why the client's legs hurt from atherosclerosis. 1. The nurse should assume the client is a layperson and should not explain disease processes using medical terminology. 2. This is passing the buck; the nurse should have the knowledge to answer this question. 3. Atherosclerosis involves the arteries, not the veins.

The client asks the nurse, "My doctor just told me that atherosclerosis is why my legs hurt when I walk. What does that mean?" Which response by the nurse would be the best response? 1. "The muscle fibers and endothelial lining of your arteries have become thickened." 2. "The next time you see your HCP, ask what atherosclerosis means." 3. "The valves in the veins of your legs are incompetent so your legs hurt." 4. "You have a hardening of your arteries that decreases the oxygen to your legs."

1. The heart tissue is dead, stress or activity may cause heart failure, and it does take about six (6) weeks for scar tissue to form. 2. The nurse should talk to the client in layperson's terms, not medical terms. Medical terminology is a foreign language to most clients. 3. This is not answering the client's question. The nurse should take any opportunity to teach the client. 4. This is a condescending response, and telling the client that he or she is not out of danger is not an appropriate response.

The client diagnosed with a myocardial infarction asks the nurse, "Why do I have to rest and take it easy? My chest doesn't hurt anymore." Which statement would be the nurse's best response? 1. "Your heart is damaged and needs about four (4) to six (6) weeks to heal." 2. "There is necrotic myocardial tissue that puts you at risk for dysrhythmias." 3. "Your doctor has ordered bedrest. Therefore, you must stay in the bed." 4. "Just because your chest doesn't hurt anymore doesn't mean you are out of danger."

3. The nurse should praise and encourage UAPs to participate in the client's care. Clients on bedrest are at risk for deep vein thrombosis, and moving the legs will help prevent this from occurring. 1. Leg movement is an appropriate action, and the UAP should not be told to stop encouraging it. 2. This behavior is not unsafe or dangerous and should not be reported to the charge nurse. 4. The nurse should praise subordinates for appropriate behavior, especially when it is helping to prevent life-threatening complications.

The client diagnosed with a myocardial infarction is on bedrest. The unlicensed assistive personnel (UAP) is encouraging the client to move the legs. Which action should the nurse implement? 1. Instruct the UAP to stop encouraging the leg movements. 2. Report this behavior to the charge nurse as soon as possible. 3. Praise the UAP for encouraging the client to move legs. 4. Take no action concerning the UAP's behavior.

3. Any neurovascular assessment data that are abnormal require intervention by the nurse; numbness may indicate decreased blood supply to the right foot. 1. After PTCA, the client must keep the right leg straight for at least six (6) to eight (8) hours to prevent any arterial bleeding from the insertion site in the right femoral artery. 2. A pressure dressing is applied to the insertion site to help prevent arterial bleeding. 4. A bounding pedal pulse indicates that adequate circulation is getting to the right foot; therefore, this would not require immediate intervention.

The client diagnosed with a myocardial infarction is six (6) hours post-right femoral percutaneous transluminal coronary angioplasty (PTCA), also known as balloon surgery. Which assessment data would require immediate intervention by the nurse? 1. The client is keeping the affected extremity straight. 2. The pressure dressing to the right femoral area is intact. 3. The client is complaining of numbness in the right foot. 4. The client's right pedal pulse is 3+ and bounding.

1. Placing a small pillow under the shoulder will prevent the shoulder from adducting toward the chest and developing a contracture. 3.The client should not ignore the paralyzed side, and the nurse must encourage the client to move it as much as possible; a written schedule may assist the client in exercising. 2.The client should be repositioned at least every two (2) hours to prevent contractures, pneumonia, skin breakdown, and other complications of immobility. 4.These exercises are recommended, but they must be done at least five (5) times a day for 10 minutes to help strengthen the muscles for walking. 5.The fingers are positioned so that they are barely flexed to help prevent contracture of the hand.

The client diagnosed with a right-sided cerebrovascular accident is admitted to the rehabilitation unit. Which interventions should be included in the nursing care plan? Select all that apply. 1. Position the client to prevent shoulder adduction. 2. Turn and reposition the client every shift. 3. Encourage the client to move the affected side. 4. Perform quadriceps exercises three (3) times a day. 5. Instruct the client to hold the fingers in a fist.

1. The nurse would anticipate an oral anticoagulant, warfarin (Coumadin), to be prescribed to help prevent thrombi formation in the atria secondary to atrial fibrillation. The thrombi can become embolic and may cause a TIA or CVA (stroke). 2.Beta blockers slow the heart rate and decrease blood pressure but would not be an anticipated medication to help prevent a TIA secondary to atrial fibrillation. 3.An anti-hyperuricemic medication is administered for a client experiencing gout and decreases the formation of tophi. 4.A thrombolytic medication is administered to dissolve a clot, and it may be ordered during the initial presentation for a client with a CVA, but not on discharge.

The client diagnosed with atrial fibrillation has experienced a transient ischemic attack (TIA). Which medication would the nurse anticipate being ordered for the client on discharge? 1. An oral anticoagulant medication. 2. A beta blocker medication. 3. An anti-hyperuricemic medication. 4. A thrombolytic medication.

2. The most probable cause of the leg cramping is potassium excretion as a result of diuretic medication. Bananas and orange juice are foods that are high in potassium. 1. The client with peripheral edema will experience calf tightness but would not have leg cramping, which is the result of low potassium levels. The timing of the diuretic will not change the side effect of leg cramping resulting from low potassium levels. 3. Weight gain is monitored in clients with CHF, and elevating the legs would de- crease peripheral edema by increasing the rate of return to the central circulation, but these interventions would not help with leg cramps. 4. Ambulating frequently and performing leg-stretching exercises will not be effective in alleviating the leg cramps.

The client diagnosed with congestive heart failure is complaining of leg cramps at night. Which nursing interventions should be implemented? 1. Check the client for peripheral edema and make sure the client takes a diuretic early in the day. 2. Monitor the client's potassium level and assess the client's intake of bananas and orange juice. 3. Determine if the client has gained weight and instruct the client to keep the legs elevated. 4. Instruct the client to ambulate frequently and perform calf-muscle stretching exercises daily.

1. Uninterrupted rest and sleep help decrease the workload of the heart and help ensure the restoration of physical and emotional health. 2. Occupational therapy addresses activities of daily living. The client should be referred to physical therapy to develop a realistic and progressive plan of activity. 3. The client with pericarditis is not usually prescribed oxygen, and 2 L/min is a low dose of oxygen that is prescribed for a client with chronic obstructive pulmonary disease (COPD). 4. Endocarditis, not pericarditis, may lead to surgery for valve replacement.

The client diagnosed with pericarditis is being discharged home. Which intervention should the nurse include in the discharge teaching? 1. Be sure to allow for uninterrupted rest and sleep. 2. Refer client to outpatient occupational therapy. 3. Maintain oxygen via nasal cannula at two (2) L/min. 4. Discuss upcoming valve replacement surgery.

3. The nurse must assess the client to determine if the pain is expected secondary to pericarditis or if the pain is indicative of a complication that requires intervention from the health-care provider. 1. Oxygen may be needed, but it is not the first intervention. 2. This would be appropriate to determine if the urine output is at least 30 mL/hr, but it is not the first intervention. 4. Using the incentive spirometer will increase the client's alveolar ventilation and help prevent atelectasis, but it is not the first intervention.

The client diagnosed with pericarditis is complaining of increased pain. Which intervention should the nurse implement first? 1. Administer oxygen via nasal cannula. 2. Evaluate the client's urinary output. 3. Assess the client for cardiac complications. 4. Encourage the client to use the incentive spirometer.

4. Tobacco use is the strongest factor in the development of atherosclerotic lesions. Nicotine decreases blood flow to the extremities and increases heart rate and blood pressure. It also increases the risk of clot formation by increasing the aggregation of platelets. 1. Being overweight is not a risk factor for atherosclerotic lesions, but it does indicate that the client does not eat a healthy diet or exercise as needed. 2. Lack of exercise is a risk factor, but it is not the strongest. 3. Although the stem did not explicitly identify diet, the nurse should assume that a client who is obese would not eat a low-fat, low-cholesterol diet.

The client diagnosed with peripheral vascular disease is overweight, has smoked two (2) packs of cigarettes a day for 20 years, and sits behind a desk all day. What is the strongest factor in the development of atherosclerotic lesions? 1. Being overweight. 2. Sedentary lifestyle. 3. High-fat, high-cholesterol diet. 4. Smoking cigarettes.

3. Stopping all activity will decrease the need of the myocardium for oxygen and may help decrease the chest pain. 1. The nurse must assume the chest pain is secondary to decreased oxygen to the myocardium and administer a sublingual nitroglycerin tablet, which is a coronary vasodilator, but this is not the first action. 2. An ECG should be ordered, but it is not the first intervention 4. Assessment is often the first nursing intervention, but when the client has chest pain and a possible MI, the nurse must first take care of the client. Taking vital signs would not help relieve chest pain.

The client diagnosed with rule-out myocardial infarction is experiencing chest pain while walking to the bathroom. Which action should the nurse implement first? 1. Administer sublingual nitroglycerin. 2. Obtain a STAT electrocardiogram. 3. Have the client sit down immediately. 4. Assess the client's vital signs.

1. Fluid intake may be restricted to reduce the cardiac workload and pressures within the heart and pulmonary circuit. 2. The head of the bed should be elevated to help improve alveolar ventilation. 3. Oxygen saturation should be no less than 93%; 90% indicates an arterial oxygen saturation of around 60 (normal is 80 to 100) 4. Total parenteral nutrition would not be prescribed for a client with mitral valve replacement. It is ordered for clients with malnutrition, gastrointestinal disorders, or conditions in which increased calories are needed, such as burns.

The client had open-heart surgery to replace the mitral valve. Which intervention should the intensive care unit nurse implement? 1. Restrict the client's fluids as ordered. 2. Keep the client in the supine position. 3. Maintain oxygen saturation at 90%. 4. Monitor the total parenteral nutrition.

4. Raising the toilet seat is modifying the home and addresses the client's weakness in being able to sit down and get up without straining muscles or requiring lifting assistance from the wife. 1.The rubber mat will stabilize the plate and prevent it from slipping away from the client learning to feed himself, but this does not address generalized weakness. 2.A long-handled bath sponge will assist the client when showering hard-to-reach areas, but it is not a home modification, nor will it help with generalized weakness. 3. Clothes with Velcro closures will make dressing easier, but they do not constitute a home modification and do not address generalized weakness.

The client has been diagnosed with a cerebrovascular accident (stroke). The client's wife is concerned about her husband's generalized weakness. Which home modification should the nurse suggest to the wife prior to discharge? 1. Obtain a rubber mat to place under the dinner plate. 2. Purchase a long-handled bath sponge for showering. 3. Purchase clothes with Velcro closure devices. 4. Obtain a raised toilet seat for the client's bathroom.

1. The nurse should monitor the vital signs for any client who has just undergone surgery. 2. A pericardiocentesis involves entering the pericardial sac. Assessing heart and lung sounds allows assessment for cardiac failure. 3. The pericardial fluid is documented as output. 4. Evaluating the client's cardiac rhythm allows the nurse to assess for cardiac failure, which is a complication of pericardiocentesis. 5. The client should be in the semi-Fowler's position, not in a flat position, which increases the workload of the heart.

The client has just had a pericardiocentesis. Which interventions should the nurse implement? Select all that apply. 1. Monitor vital signs every 15 minutes for the first hour. 2. Assess the client's heart and lung sounds. 3. Record the amount of fluid removed as output. 4. Evaluate the client's cardiac rhythm. 5. Keep the client in the supine position.

3. Aspirin and nonsteroidal anti- inflammatory drugs (NSAIDs) interfere with clotting and may potentiate the effects of the anticoagulant therapy, which the client with a mechanical valve will be prescribed. Therefore, the client should not take aspirin daily. 1. Prophylactic antibiotics before invasive procedures prevent infectious endocarditis. 2. The client is undergoing anticoagulant therapy and should use a soft-bristle toothbrush to help prevent gum trauma and bleeding. 4. The client should alternate rest with activity to prevent fatigue to help decrease the workload of the heart.

The client has just received a mechanical valve replacement. Which behavior by the client indicates the client needs more teaching? 1. The client takes prophylactic antibiotics. 2. The client uses a soft-bristle toothbrush. 3. The client takes an enteric-coated aspirin daily. 4. The client alternates rest with activity.

2. Rheumatic fever, a systemic inflammatory disease caused by an abnormal immune response to pharyngeal infection by group A beta-hemolytic streptococci, causes carditis in about 50% of people who develop it. 1. A sore throat in the last month would not support the diagnosis of carditis. 3. Carditis is not a genetic or congenital disease process. 4. This is an appropriate question to ask any client, but OTC medications do not cause carditis.

The client is admitted to the medical unit to rule out carditis. Which question should the nurse ask the client during the admission interview to support this diagnosis? 1. "Have you had a sore throat in the last month?" 2. "Did you have rheumatic fever as a child?" 3. "Do you have a family history of carditis?" 4. "What over-the-counter (OTC) medications do you take?"

1.The client with CHF would exhibit tachycardia (apical pulse rate of 110), dependent edema, fatigue, third heart sounds, lung congestion, and change in mental status. 2. The client with CHF usually has pink frothy sputum and crackles that do not clear with coughing. 3. The client with CHF would report sleeping on at least two pillows, if not sleeping in an upright position, and labored breathing, not eupnea, which means normal breathing. 4. In a client diagnosed with heart failure, the apical pulse, not the radial pulse, is the best place to assess the cardiac status.

The client is admitted to the telemetry unit diagnosed with acute exacerbation of congestive heart failure (CHF). Which signs/symptoms would the nurse expect to find when assessing this client? 1. Apical pulse rate of 110 and 4+ pitting edema of feet. 2. Thick white sputum and crackles that clear with cough. 3. The client sleeping with no pillow and eupnea. 4. Radial pulse rate of 90 and capillary refill time <3 seconds.

4. Rheumatic heart disease is the most common cause of valvular heart disease. 1. An acute myocardial infarction can dam- age heart valves, causing tearing, ischemia, or damage to heart muscles that affects valve leaflet function, but coronary heart disease does not cause valvular heart disease. 2. Valvular heart disease does not show a genetic etiology. 3. Smoking can cause coronary artery disease, but it does not cause valvular heart disease.

The client is being evaluated for valvular heart disease. Which information would be most significant? 1. The client has a history of coronary artery disease. 2. There is a family history of valvular heart disease. 3. The client has a history of smoking for 10 years. 4. The client has a history of rheumatic heart disease.

1. Acute pericardial effusion interferes with normal cardiac filling and pump- ing, causing venous congestion and decreased cardiac output. Muffled heart sounds, indicative of acute pericarditis, must be reported to the health-care provider. 2. Distended, not non-distended, jugular veins would warrant immediate intervention. 3. Decreasing quality of peripheral pulses, not bounding peripheral pulses, would warrant immediate intervention. 4. A pericardial friction rub is a classic symptom of acute pericarditis, but it would not warrant immediate intervention.

The client is diagnosed with acute pericarditis. Which sign/symptom warrants immediate attention by the nurse? 1. Muffled heart sounds. 2. Nondistended jugular veins. 3. Bounding peripheral pulses. 4. Pericardial friction rub.

2. Expressive aphasia means that the client cannot communicate thoughts but understands what is being communicated; this leads to frustration, anger, depression, and the inability to verbalize needs, which, in turn, causes the client to have a lack of control and feel powerless 1. Potential for injury is a physiological problem, not a psychosocial problem. 3. A disturbance in thought processes is a cognitive problem; with expressive aphasia the client's thought processes are intact. 4. Sexual dysfunction can have a psychosocial component or a physical component, but it is not related to expressive aphasia.

The client is diagnosed with expressive aphasia. Which psychosocial client problem would the nurse include in the plan of care? 1. Potential for injury. 2. Powerlessness. 3. Disturbed thought processes. 4. Sexual dysfunction.

4. Chest pain is the most common symptom of pericarditis, usually has an abrupt onset, and is aggravated by respiratory movements (deep inspiration, coughing), changes in body position, and swallowing. 1. Pulsus paradoxus is the hallmark of cardiac tamponade; a paradoxical pulse is markedly decreased in amplitude during inspiration. 2. Fatigue and arthralgias are nonspecific signs/symptoms that usually occur with myocarditis. 3. Petechiae on the trunk, conjunctiva, and mucous membranes and hemorrhagic streaks under the fingernails or toenails occur with endocarditis.

The client is diagnosed with pericarditis. Which are the most common signs/symptoms the nurse would expect to find when assessing the client? 1. Pulsus paradoxus. 2. Complaints of fatigue and arthralgias. 3. Petechiae and splinter hemorrhages. 4. Increased chest pain with inspiration.

2. The nurse must always assess the client to determine if the chest pain that is occurring is expected postoperatively or if it is a complication of the surgery. 1. The nurse should medicate the client as needed, but it is not the first intervention. 3. Turning will help decrease complications from immobility, such as pneumonia, but it will not help relieve the client's pain. 4. The nurse, not a machine, should always take care of the client.

The client is one (1) day postoperative coronary artery bypass surgery. The client complains of chest pain. Which intervention should the nurse implement first? 1. Medicate the client with intravenous morphine. 2. Assess the client's chest dressing and vital signs. 3. Encourage the client to turn from side to side. 4. Check the client's telemetry monitor.

2. The nurse must make sure that blood is circulating to the right leg, so the client should be assessed for pulses, paresthesia, paralysis, coldness, and pallor. 1. The client's right leg should be kept straight to prevent arterial bleeding from the femoral insertion site for the catheter used to perform the catheterization. 3. The head of the bed should be elevated no more than 10 degrees. The client should be kept on bedrest, flat with the affected extremity straight, to help decrease the chance of femoral artery bleeding. 4. The gag reflex is assessed if a scope is in- serted down the trachea (bronchoscopy) or esophagus (endoscopy) because the throat is numbed when inserting the scope. A catheter is inserted in the femoral or brachial artery when performing a cardiac catheterization.

The client is scheduled for a right femoral cardiac catheterization. Which nursing intervention should the nurse implement after the procedure? 1. Perform passive range-of-motion exercises. 2. Assess the client's neurovascular status. 3. Keep the client in high Fowler's position. 4. Assess the gag reflex prior to feeding the client.

2. The client needs to be taught ways to lower the cholesterol level. 1.The cholesterol level should be less than 200 mg/dL. 3. The client should be taught a low-fat, low-cholesterol diet to help lower the cholesterol level. 4. The nurse needs to discuss facts concern- ing the cholesterol level and teach the client. A therapeutic conversation would not be appropriate.

The client tells the nurse that his cholesterol level is 240 mg/dL. Which action should the nurse implement? 1. Praise the client for having a normal cholesterol level. 2. Explain that the client needs to lower the cholesterol level. 3. Discuss dietary changes that could help increase the level. 4. Allow the client to ventilate feelings about the blood test result.

3. Cardiac rehabilitation is the most appropriate referral. The client can start rehabilitation in the hospital and then attend an outpatient cardiac rehabilitation clinic, which includes progressive exercise, diet teaching, and classes on modifying risk factors. 1. The social worker addresses financial concerns or referrals after discharge, which are not indicated for this client. 2. Physical therapy addresses gait problems, lower extremity strength building, and assisting with transfer, which are not required for this client. 4. Occupational therapy assists the client in regaining activities of daily living and covers mainly fine motor activities.

The client who has had a myocardial infarction is admitted to the telemetry unit from intensive care. Which referral would be most appropriate for the client? 1. Social worker. 2. Physical therapy. 3. Cardiac rehabilitation. 4. Occupational therapy.

4. In this invasive procedure, performed in a cardiac catheterization laboratory, the client has a catheter inserted into the femoral artery. Therefore, the client must keep the leg straight to pre- vent hemorrhaging at the insertion site. 1. Percutaneous balloon valvuloplasty is not an open-heart surgery; therefore, the chest will not be open and the client will not have a chest tube. 2. This is not an open-heart surgery; there- fore, the client will not have a chest dressing. 3. The endotracheal (ET) tube is inserted if the client is on a ventilator, and this surgery does not require putting the client on a ventilator.

The client who has just had a percutaneous balloon valvuloplasty is in the recovery room. Which intervention should the recovery room nurse implement? 1. Assess the client's chest tube output. 2. Monitor the client's chest dressing. 3. Evaluate the client's endotracheal (ET) lip line. 4. Keep the client's affected leg straight.

1. The client is at risk for developing endocarditis and should take prophylactic antibiotics before any invasive procedure. 2. Antibiotics have nothing to do with how much the teeth bleed during a cleaning. 3. Teeth cleaning will not cause the valve to malfunction. 4. Vegetation develops on valves secondary to bacteria that cause endocarditis, but the client may not understand "vegetative growth on your valves"; therefore, this is not the most appropriate answer.

The client with a mechanical valve replacement asks the nurse, "Why do I have to take antibiotics before getting my teeth cleaned?" Which response by the nurse is most appropriate? 1. "You are at risk of developing an infection in your heart." 2. "Your teeth will not bleed as much if you have antibiotics." 3. "This procedure may cause your valve to malfunction." 4. "Antibiotics will prevent vegetative growth on your valves."

1. This is a correct statement presented in layman's terms. When the coronary arteries cannot supply adequate oxygen to the heart muscle, there is chest pain. 2. This is the explanation in medical terms that should not be used when explaining medical conditions to a client. 3. This explains congestive heart failure but does not explain why chest pain occurs. 4. Respiratory compromise occurs when the lungs cannot oxygenate the blood, such as occurs with altered level of consciousness, cyanosis, and increased respiratory rate.

The client with coronary artery disease asks the nurse, "Why do I get chest pain?" Which statement would be the most appropriate response by the nurse? 1. "Chest pain is caused by decreased oxygen to the heart muscle." 2. "There is ischemia to the myocardium as a result of hypoxemia." 3. "The heart muscle is unable to pump effectively to perfuse the body." 4. "Chest pain occurs when the lungs cannot adequately oxygenate the blood."

1. The Holter monitor is a 24-hour electrocardiogram, and the client must keep an accurate record of activity so that the health-care provider can compare the ECG recordings with different levels of activity. 2. The Holter monitor should not be removed for any reason. 3. All medications should be taken as prescribed. 4. The client should perform all activity as usual while wearing the Holter monitor so the HCP can get an accurate account of heart function during a 24-hour period.

The client with coronary artery disease is prescribed a Holter monitor. Which intervention should the nurse implement? 1. Instruct client to keep a diary of activity, especially when having chest pain. 2. Discuss the need to remove the Holter monitor during a.m. care and showering. 3. Explain that all medications should be withheld while wearing a Holter monitor. 4. Teach the client the importance of decreasing activity while wearing the monitor.

2. Blood cultures must be done before administering antibiotics so that an ad- equate number of organisms can be obtained to culture and identify. 1. The nurse must obtain blood cultures prior to administering antibiotics. 3. An echocardiogram allows visualization of vegetations and evaluation of valve function. However, antibiotic therapy is priority before diagnostic tests, and blood cultures must be obtained before administering medication. 4. Bedrest should be implemented, but the first intervention should be obtaining blood cultures so that antibiotic therapy can be started as soon as possible.

The client with infective endocarditis is admitted to the medical department. Which health-care provider's order should be implemented first? 1. Administer intravenous antibiotic. 2. Obtain blood cultures times two (2). 3. Schedule an echocardiogram. 4. Encourage bedrest with bathroom privileges.

3. NSAIDs must be taken with food, milk, or antacids to help decrease gastric dis- tress. NSAIDs reduce fever, inflammation, and pericardial pain. 1. Steroids, such as prednisone, not NSAIDs, must be tapered off to prevent adrenal insufficiency. 2. NSAIDs will not make clients drowsy 4. NSAIDs should be taken regularly around the clock to help decrease inflammation, which, in turn, will decrease pain.

The client with pericarditis is prescribed a nonsteroidal anti-inflammatory drug (NSAID). Which teaching instruction should the nurse discuss with the client? 1. Explain the importance of tapering off the medication. 2. Discuss that the medication will make the client drowsy. 3. Instruct the client to take the medication with food. 4. Tell the client to take the medication when the pain level is around "8."

2. an INR of 1.2 is not within the expected therapeutic ranges of 2 to 3 and indicates a need for an increase in warfarin dosage

The clinic nurse obtains this information about a client who is taking warfarin after having DVT. Which finding is most indicative of a need for a change in therapy? 1. BP 106/54 2. INR 1.2 3. bruises are noted at sites where blood has been drawn 4. client reports eating green salad for lunch everyday

3. Sexual activity is a risk factor for angina resulting from coronary artery disease. The client's being elderly should not affect the nurse's assessment of the client's concerns about sexual activity. 1. Bowel movements are important, but they are not pertinent to coronary artery disease. 2. Chest x-rays are usually done for respiratory problems, not for coronary artery disease. 4. Weight change is not significant in a client with coronary artery disease.

The elderly client has coronary artery disease. Which question should the nurse ask the client during the client teaching? 1. "Do you have a daily bowel movement?" 2. "Do you get yearly chest x-rays?" 3. "Are you sexually active?" 4. "Have you had any weight change?"

3. Even with antibiotic treatment for rheumatic fever, the client may experience bacterial endocarditis in later years and should know this may occur. 1. The full course of antibiotics must be taken to help ensure complete destruction of streptococcal infection. 2. Antibiotics kill bacteria but also destroy normal body flora in the vagina, bowel, and mouth, leading to a superinfection. 4. A throat culture is taken to diagnose group A beta-hemolytic streptococcus and is positive in 25% to 40% of clients with acute rheumatic fever.

The female client is diagnosed with rheumatic fever and prescribed penicillin, an antibiotic. Which statement indicates the client needs more teaching concerning the discharge teaching? 1. "I must take all the prescribed antibiotics." 2. "I may get a vaginal yeast infection with penicillin." 3. "I will have no problems as long as I take my medication." 4. "My throat culture was positive for a streptococcal infection."

2. Orthostatic hypotension may occur with ACE inhibitors as a result of vasodilation. Therefore, the nurse should in- struct the client to rise slowly and sit on the side of the bed until equilibrium is restored. 1. If a cough develops, the client should notify the health-care provider because this is an adverse reaction and the HCP will discontinue the medication. 3. ACE inhibitors may cause the client to retain potassium; therefore, the client should not increase potassium intake. 4. An ACE inhibitor should be taken one (1) hour before meals or two (2) hours after a meal to increase absorption of the medication.

The health-care provider has ordered an angiotensin-converting enzyme (ACE) inhibitor for the client diagnosed with congestive heart failure. Which discharge instructions should the nurse include? 1. Instruct the client to take a cough suppressant if a cough develops. 2. Teach the client how to prevent orthostatic hypotension. 3. Encourage the client to eat bananas to increase potassium level. 4. Explain the importance of taking the medication with food.

1. An S3 indicates left ventricular failure and should be reported to the health- care provider. It is a potential life- threatening complication of a myocardial infarction. 2. Elevating the head of the bed will not do anything to help a failing heart. 3. This is not a normal finding; it indicates heart failure. 4. Morphine is administered for chest pain, not for heart failure, which is suggested by the S3 sound.

The intensive care department nurse is assessing the client who is 12 hours post-myocardial infarction. The nurse assesses an S3 heart sound. Which intervention should the nurse implement? 1. Notify the health-care provider immediately. 2. Elevate the head of the client's bed. 3. Document this as a normal and expected finding. 4. Administer morphine intravenously.

3.This action is inappropriate and would require intervention by the nurse because pulling on a flaccid shoulder joint could cause shoulder dislocation; the client should be pulled up by placing the arm underneath the back or using a lift sheet. 1.Placing a gait belt prior to ambulating is an appropriate action for safety and would not require the nurse to intervene. 2.Placing the client in a supine position with the head turned to the side is not a problem position, so the nurse does not need to intervene. 4.The client should be encouraged and praised for attempting to perform any activities independently, such as combing hair or brushing teeth.

The nurse and an unlicensed assistive personnel (UAP) are caring for a client with right-sided paralysis. Which action by the UAP requires the nurse to intervene? 1. The assistant places a gait belt around the client's waist prior to ambulating. 2. The assistant places the client on the back with the client's head to the side. 3. The assistant places a hand under the client's right axilla to move up in bed. 4. The assistant praises the client for attempting to perform ADLs independently.

4. The UAP can assist the x-ray technician in positioning the client for the portable x-ray. This does not require judgment. 1. Allowing the UAP to take a client down to smoke is not cost effective and is not sup- portive of the medical treatment regimen that discourages smoking. 2. The client going to the ICU would be unstable, and the nurse should not dele- gate to a UAP any nursing task that in- volves an unstable client. 3. The nurse cannot delegate teaching.

The nurse and an unlicensed assistive personnel (UAP) are caring for four clients on a telemetry unit. Which nursing task would be best for the nurse to delegate to the UAP? 1. Assist the client to go down to the smoking area for a cigarette. 2. Transport the client to the intensive care unit via a stretcher. 3. Provide the client going home discharge-teaching instructions. 4. Help position the client who is having a portable x-ray done.

4.the most common complication after coronary arteriography is a hemorrhage and the earliest sign of a hemorrhage is an increased HR

The nurse assesses a pt who has just returned to the recovery area after a coronary arteriography. Which info is of most concern? 1.BP 154/78 2.Pedal pulse are palpable +1 3.L groin has 3cm bruise 4.apical pulse is 122 and regular

C. monounsaturated fats should be consumed

The nurse determines that teaching about implementing dietary changes to decrease the risk of CAD has been effective when the pt says: a. "I should not eat any red meat such as beef, pork, or lamb" b. I should have some type of fish at least 3 times a week" c. "most of my fat intake should be from olive oil or the oils in nuts" d. "If i reduce the fat in my diet to about 25% of my cals, I will be healthier"

4. The nurse must first put the client in a sitting position to decrease the workload of the heart by decreasing venous return and maximizing lung expansion. Then, the nurse could take vital signs and check the pulse oximeter and then sponge the client's forehead. 1. Sponging the client's forehead would be appropriate, but it is not the first intervention. 2. Obtaining a pulse oximeter reading would be appropriate, but it is not the first intervention. 3. Taking the vital signs would be appropriate, but it is not the first intervention.

The nurse enters the room of the client diagnosed with congestive heart failure. The client is lying in bed gasping for breath, is cool and clammy, and has buccal cyanosis. Which intervention would the nurse implement first? 1. Sponge the client's forehead. 2. Obtain a pulse oximetry reading. 3. Take the client's vital signs. 4. Assist the client to a sitting position.

4. The client's blood pressure is low, and a calcium channel blocker could cause the blood pressure to bottom out. 1. The apical pulse is within normal limits— 60 to 100 beats per minute. 2. The serum calcium level is not monitored when calcium channel blockers are given. 3. Occasional PVCs would not warrant immediate intervention prior to administering this medication.

The nurse is administering a calcium channel blocker to the client diagnosed with a myocardial infarction. Which assessment data would cause the nurse to question administering this medication? 1. The client's apical pulse is 64. 2. The client's calcium level is elevated. 3. The client's telemetry shows occasional PVCs. 4. The client's blood pressure is 90/62.

2.The most common motor dysfunction of a CVA is paralysis of one side of the body, hemiplegia; in this case with a left-sided CVA, the paralysis would affect the right side. Ataxia is an impaired ability to coordinate movement. 1.A left-sided CVA will result in right-sided motor deficits; hemiparesis is weakness of one half of the body, not just the upper extremity. Apraxia, the inability to perform a previously learned task, is a communication loss, not a motor loss. 3.Homonymous hemianopsia (loss of half of the visual field of each eye) and diplopia (double vision) are visual field deficits that a client with a CVA may experience, but they are not motor losses. 4.Personality disorders occur in clients with a right-sided CVA and are cognitive deficits; hostility is an emotional deficit.

The nurse is assessing a client experiencing motor loss as a result of a left-sided cere- brovascular accident (CVA). Which clinical manifestations would the nurse document? 1. Hemiparesis of the client's left arm and apraxia. 2. Paralysis of the right side of the body and ataxia. 3. Homonymous hemianopsia and diplopia. 4. Impulsive behavior and hostility toward family.

1. BNP is a specific diagnostic test. Levels higher than normal indicate congestive heart failure, with the higher the number, the more severe the CHF. 2. An elevated CK-MB would indicate a myocardial infarction, not severe CHF. CK-MB is an isoenzyme. 3. A positive D-dimer would indicate a pul- monary embolus. 4. A positive ventilation/perfusion (V/Q) scan (ratio) would indicate a pulmonary embolus.

The nurse is assessing the client diagnosed with congestive heart failure. Which laboratory data would indicate that the client is in severe congestive heart failure? 1. An elevated B-type natriuretic peptide (BNP). 2. An elevated creatine kinase (CK-MB). 3. A positive D-dimer. 4. A positive ventilation/perfusion (V/Q) scan.

3. Being able to perform activities of daily living (ADLs) without shortness of breath (dyspnea) would indicate the client's condition is improving. The client's heart is a more effective pump and can oxygenate the body better with- out increasing fluid in the lungs. 1. Pitting edema changing from 3+ to 4+ indi- cates a worsening of the CHF. 2. The client's ability to take the radial pulse would evaluate teaching, not medical treatment. 4. Any jugular vein distention indicates that the right side of the heart is failing, which would not indicate effective medical treatment.

The nurse is assessing the client diagnosed with congestive heart failure. Which signs/symptoms would indicate that the medical treatment has been effective? 1. The client's peripheral pitting edema has gone from 3+ to 4+. 2. The client is able to take the radial pulse accurately. 3. The client is able to perform ADLs without dyspnea. 4. The client has minimal jugular vein distention.

2. Aspirin is an antiplatelet medication and should be administered orally. 3. Oxygen will help decrease myocardial ischemia, thereby decreasing pain 1. Morphine should be administered intravenously, not intramuscularly 4. The supine position will increase respiratory effort, which will increase myocardial oxygen consumption; the client should be in the semi-Fowler's position. 5. Nitroglycerin, a coronary vasodilator, is administered sublingually, not subcutaneously.

The nurse is caring for a client diagnosed with a myocardial infarction who is experiencing chest pain. Which interventions should the nurse implement? Select all that apply. 1. Administer morphine intramuscularly. 2. Administer an aspirin orally. 3. Apply oxygen via a nasal cannula. 4. Place the client in a supine position. 5. Administer nitroglycerin subcutaneously.

3. hyperkalemia is a common AE of both ACE inhibs and K+ sparing diuretics

The nurse is caring for a hospitalized client with HF who is receiving captopril and spironolactone. Which lab will be most important to monitor? 1. sodium 2.glucose 3.K+ 4. alkaline phosphatase level

C

The nurse is caring for a patient that is receiving HCTZ 25 mg PO daily for HTN. Which of these meds would indicate to the nurse that the patient may be developing dehydration? a. postural hypotension b. bradycardia c. hyperkalemia d. increased respiratory rate

A

The nurse is caring for a patient with expressive aphasia. Which should the nurse include as part of long-range planning for the patient? a. Provide positive feedback when the patient uses a word correctly. b. Suggest that the patient get assistance at home because the disability is permanent. c. Communicate with "yes" and "no" questions only. d. Support the family to accept the fact that the patient cannot participate in verbal communication.

C. see if antibiotics are working

The nurse is caring for a pt admitted with heart failure secondary to infective endocarditis. Which intervention would be the priority? a. encourage caregivers t learn CPR b. consider a consultation with hospice for palliative care c. monitor the patient's response to prescribe meds d. arrange for the patient to enter a cardiac rehabilitation program

d. orthostatic hypotension

The nurse is caring for a pt that has been prescribed beta blockers to manage angina. Which of the following is essential teaching? a. you should wear support hose continuously to minimize venous stasis b. lie down from 30 minutes after taking the med c. avoid tasks that require energy expenditures d. sit on the edge of the bed a short time before arising

C

The nurse is caring for apt who survived a sudden cardiac death. What should the nurse include in the discharge instructions? a. "because you responded well to CPR, ou will not need an implanted defibrillator" b. "the most common way to prevent another arrest is take your prescribed drugs" c. "your family members should learn how to perform CPR and practices these skills regularly" d. "since there was not evidence of a heart attack, you do not need to worry about another episode"

2. The UAP can feed a client. 1. Teaching cannot be delegated to a UAP. 3. The UAP cannot assess the client and does not have the education to interpret laboratory data. 4. A unit of blood must be checked by two (2) registered nurses at the bedside.

The nurse is caring for clients on a telemetry floor. Which nursing task would be most appropriate to delegate to an unlicensed assistive personnel (UAP)? 1. Teach the client how to perform a glucometer check. 2. Assist feeding the client diagnosed with congestive heart failure. 3. Check the cholesterol level for the client diagnosed with atherosclerosis. 4. Assist the nurse to check the unit of blood at the client's bedside.

2. The client should not take digoxin if the radial pulse is less than 60. 3. The client should be on a low-sodium diet to prevent water retention. 1. The client should notify the HCP of weight gain of more than two (2) or three(3) pounds in one (1) day. 4. The color of the urine should not change to a dark color; if anything, it might become lighter and the amount will increase with diuretics. 5. Instruct the client to take the diuretic in the morning to prevent nocturia.

The nurse is developing a discharge-teaching plan for the client diagnosed with congestive heart failure. Which interventions should be included in the plan? Select all that apply. 1. Notify health-care provider of a weight gain of more than one (1) pound in a week. 2. Teach client how to count the radial pulse when taking digoxin, a cardiac glycoside. 3. Instruct client to remove the saltshaker from the dinner table. 4. Encourage client to monitor urine output for change in color to become dark. 5. Discuss the importance of taking the loop diuretic furosemide at bedtime.

2. Audible S1 and S2 sounds are normal for a heart with adequate output. An audible S3 sound might indicate left ventricular failure which could be life threatening. 1. Ambulating in the hall by day of discharge would be a more appropriate goal for an activity-intolerance nursing diagnosis. 3. This is a nursing intervention, not a short-term goal, for this client. 4. A pulse oximeter reading would be a goal for impaired gas exchange, not for cardiac output.

The nurse is developing a nursing care plan for a client diagnosed with congestive heart failure. A nursing diagnosis of "decreased cardiac output related to inability of the heart to pump effectively" is written. Which short-term goal would be best for the client? The client will: 1. Be able to ambulate in the hall by date of discharge. 2. Have an audible S1 and S2 with no S3 heard by end of shift. 3. Turn, cough, and deep breathe every two (2) hours. 4. Have a pulse oximeter reading of 98% by day two (2) of care.

3. developing a plan of care is the job of the RN

The nurse is developing standardized care plan for the post op care of pts undergoing cardiac surgery. The unit is staffed with RNs. LPN/LVN, and UAP. Which nursing activity will need to be performed by RN staff members? 1.removing chest and leg dressings on the second post op day and cleaning the incisions with antibacterial swabs 2. reinforcing client and family teaching about how the need to deep breathe and cough at least every 2 hr while awake 3. developing an individual plan for dishcarge teaching based on discharge meds and needed lifestyle changes 4. administering oral analgesic meds as needed before helping the client out of bed

2. Stopping the activity decreases the heart's need for oxygen and may help decrease the angina (chest pain). 1. The client should take the coronary vasodilator nitroglycerin sublingually, but it is not the first intervention. 3. The client should keep a diary of when angina occurs, what activity causes it, and how many nitroglycerin tablets are taken before chest pain is relieved. 4. If the chest pain (angina) is not relieved with three (3) nitroglycerin tablets, the client should call 911 or have someone take him to the emergency department. Notifying the HCP may take too long.

The nurse is discussing angina with a client who is diagnosed with coronary artery disease. Which action should the client take first when experiencing angina? 1. Put a nitroglycerin tablet under the tongue. 2. Stop the activity immediately and rest. 3. Document when and what activity caused angina. 4. Notify the health-care provider immediately.

3. When it is cold outside, vasoconstriction occurs, and this will decrease oxygen to the heart muscle. Therefore, the client should not exercise when it is cold outside. 1. Isometric exercises are weight lifting-type exercises. A client with CAD should per- form isotonic exercises, which increase muscle tone, not isometric exercises. 2. The client should walk at least 30 minutes a day to increase collateral circulation. 4. The client should wear good supportive tennis shoes when ambulating, not sandals or other open-toed shoes.

The nurse is discussing the importance of exercise with the client diagnosed with coronary artery disease. Which intervention should the nurse implement? 1. Perform isometric exercises daily. 2. Walk for 15 minutes three (3) times a week. 3. Do not walk outside if it is less than 40 ̊F. 4. Wear open-toed shoes when ambulating.

1. Collateral circulation is the development of blood supply around narrowed arteries; it helps prevent complications of atherosclerosis, including myocardial infarction, cerebrovascular accidents, and peripheral vascular disease. Exercise promotes the development of collateral circulation. 2. Isometric (weight-lifting) exercises help develop muscle mass, but this type of exercise does not help decrease complications of atherosclerosis. 3. A low-fat, low-cholesterol diet may help decrease the plaque formation, but exercise will not do this. 4. Isotonic exercises, such as walking and swimming, promote the movement of glucose across the cell membrane, but this is not why such exercises are recommended for prevention of atherosclerotic complications.

The nurse is discussing the importance of exercising with a client who is diagnosed with CAD. Which statement best describes the scientific rationale for encouraging 30 minutes of walking daily to help prevent complications of atherosclerosis? 1. Exercise promotes the development of collateral circulation. 2. Isometric exercises help develop the client's muscle mass. 3. Daily exercise helps prevent plaque from developing in the vessel. 4. Isotonic exercises promote the transport of glucose into the cell.

1. A normal HDL level is good because HDL transports cholesterol away from the tissues and cells of the arterial wall to the liver for excretion. This helps decrease the development of atherosclerosis. 2. The normal HDL level was the result of a test measuring high-density lipoproteins, not free fatty acids and glycerol in the blood, which are measured by the serum triglyceride level. Triglycerides are a source of energy. 3. Low-density lipoproteins (LDLs), not HDLs, are the primary transporters of cholesterol into the cell. They have the harmful effect of depositing cholesterol into the walls of the arterial vessels. 4. A normal HDL level is good and the client does not need to change the diet.

The nurse is discussing the pathophysiology of atherosclerosis with a client who has a normal high-density lipoprotein (HDL) level. Which information should the nurse discuss with the client concerning HDL? 1. A normal HDL is good because it has a protective action in the body. 2. HDL lipoprotein level measures the free fatty acids and glycerol in the blood. 3. HDLs are the primary transporters of cholesterol into the cell. 4. The client needs to decrease the amount of cholesterol and fat in the diet.

3. premature vent contraction occurring in the setting of acute myocardial injury or infarction can lead to vent tachycardia and/or vent fibrillation (cardiac arrest), best choice because its life threatening

The nurse is monitoring the cardiac rhythms of pts in the coronary care unit. What pt will need immediate intervention? 1. pt admitted with HF who has atrial fibrillation with a pulse of 88 while at rest 2. pt with newly implanted demand vent pacemaker who has occasional periods of sinus rhythm and pulse of 90-100 3. pt who has had an acute MI and has sinus rhythm and pulse of 76 with frequent premature vent contractions 4.pt who recently started taking atenolol and has a first degree heart block with a pulse of 58

3. communication is key when working as a team

The nurse is participating as a team member in the resuscitation of a pt who has has a cardiac arrest. The HCP who is directing the resuscitation asks the nurse to administer epi 1mg IV. After giving the med, which action should the nurse take next? 1.prepare to defibrillate the pt 2. offer to take over chest compressions 3.state: "epi 1 mg IV has been given" 4. continue to monitor the pt's responsiveness

4. A collaborative intervention is an intervention in which another health-care discipline—in this case, occupational therapy—is used in the care of the client. 1.Agnosia is the failure to recognize familiar objects; therefore, observing the client for possible aspiration is not appropriate. 2.A semi-Fowler's position is appropriate for sleeping, but agnosia is the failure to recognize familiar objects; therefore, this intervention is inappropriate. 3.Placing suction at the bedside will help if the client has dysphagia (difficulty swallowing), not agnosia, which is failure to recognize familiar objects.

The nurse is planning care for a client experiencing agnosia secondary to a cerebrovas- cular accident. Which collaborative intervention will be included in the plan of care? 1. Observe the client swallowing for possible aspiration. 2. Position the client in a semi-Fowler's position when sleeping. 3. Place a suction setup at the client's bedside during meals. 4. Refer the client to an occupational therapist for evaluation.

2. A beta blocker decreases sympathetic stimulation to the heart, thereby de- creasing the heart rate. An apical rate less than 60 indicates a lower-than- normal heart rate and should make the nurse question administering this medication because it will further decrease the heart rate. 1. This blood pressure is normal and the nurse would administer the medication. 3. A headache will not affect administering the medication to the client. 4. The potassium level is within normal lim- its, but it is usually not monitored prior to administering a beta blocker.

The nurse is preparing to administer a beta blocker to the client diagnosed with coronary artery disease. Which assessment data would cause the nurse to question administering the medication? 1. The client has a BP of 110/70. 2. The client has an apical pulse of 56. 3. The client is complaining of a headache. 4. The client's potassium level is 4.5 mEq/L.

4. anticoagulant meds are high alert meds and require safeguards

The nurse is preparing to administer the following meds to a client with multiple health problems who has been hospitalized with DVT. Which med is most important to double check with another licensed nurse? 1.Famotidine 20 mg IV 2. Furosemide 40 mg IV 3. Digoxin 0.25 mg PO 4. Warfarin 2.5 mg PO

1. The therapeutic range for most clients' INR is 2 to 3, but for a client with a mechanical valve replacement it is 2 to 3.5. The medication should be given as ordered and not withheld. 2. Vitamin K is the antidote for an overdose of warfarin, but 2.7 is within therapeutic range. 3. This laboratory result is within the therapeutic range, INR 2 to 3, and the medication does not need to be withheld. 4. There is no need for the nurse to assess for bleeding because 2.7 is within therapeutic range.

The nurse is preparing to administer warfarin (Coumadin), an oral anticoagulant, to a client with a mechanical valve replacement. The client's international normalized ratio (INR) is 2.7. Which action should the nurse implement? 1. Administer the medication as ordered. 2. Prepare to administer vitamin K (AquaMephyton). 3. Hold the medication and notify the HCP. 4. Assess the client for abnormal bleeding.

2.this statement acknowledges the client's statement and asks for further clarification

The nurse is preparing to implement teaching about a heart healthy diet and activity levels for a client who has had a MI and a client's spouse. The pt says "I don't see why I need any teaching. I don't think I need to change anything right now." Which response is most appropriate? 1."do you think your famil may want you to make some lifestyle changes?" 2."can you tell me why you feel you don;t feel that you need to make any changes?" 3. "you are still in a stage of denial, but you will want this info later on" 4. "even though you don't want to chnage, it's imporatnt that you have this teaching"

3. This is the scientific rationale for why diabetes mellitus is a modifiable risk factor for atherosclerosis. 1. Glucose does not combine with carbon monoxide. 2. Vasoconstriction is not a risk factor for developing atherosclerosis. 4. When glucose combines with the hemoglobin in a laboratory test called glycosylated hemoglobin, the result can determine the client's average glucose level over the past three (3) months.

The nurse is teaching a class on atherosclerosis. Which statement describes the scientific rationale as to why diabetes is a risk factor for developing atherosclerosis? 1. Glucose combines with carbon monoxide, instead of with oxygen, and this leads to oxygen deprivation of tissues. 2. Diabetes stimulates the sympathetic nervous system, resulting in peripheral constriction that increases the development of atherosclerosis. 3. Diabetes speeds the atherosclerotic process by thickening the basement membrane of both large and small vessels. 4. The increased glucose combines with the hemoglobin, which causes deposits of plaque in the lining of the vessels.

1. A modifiable risk factor is a risk factor that can possibly be altered by modify- ing or changing behavior, such as developing new ways to deal with stress. 2. The client cannot do anything about getting older, so it cannot be modified. 3. Gender is a risk factor that cannot be changed. 4. Having a family history of coronary artery disease predisposes the client to a higher risk, but this cannot be changed by the client.

The nurse is teaching a class on coronary artery disease. Which modifiable risk factor should the nurse discuss when teaching about atherosclerosis? 1. Stress. 2. Age. 3. Gender. 4. Family history.

4. Biological valves deteriorate and need to be replaced frequently; this is a dis- advantage of them. Mechanical valves do not deteriorate and do not have to be replaced often. 1. An advantage of having a biological valve replacement is that no anticoagulant therapy is needed. Anticoagulant therapy is needed with a mechanical valve replacement. 2. This is an advantage of having a biological valve replacement; infections are harder to treat in clients with mechanical valve replacement. 3. This is an advantage of having a biological valve replacement; there is a high inci- dence of thromboembolism in clients with mechanical valve replacement.

The nurse is teaching a class on valve replacements. Which statement identifies a disadvantage of having a biological tissue valve replacement? 1. The client must take lifetime anticoagulant therapy. 2. The client's infections are easier to treat. 3. There is a low incidence of thromboembolism. 4. The valve has to be replaced frequently.

1. measuring BPs are within a UAP's scope of practice

The nurse is working in an outpatient clinic where may vascular diagnostic tests are performed. Which task associated with vascular testing is most appropriate to delegate to experienced unlicensed assistive personnel (UAP)? 1. measuring ankle and brachial pressures in the pt whom the ankle-brachial index is to be calculated 2.Checking BP and pulse q10 minutes in a pt who is undergoing exercise testing 3. obtaining info about allergies from a pt who is scheduled for a left leg contrast venography 4.providing brief pt teaching for a pt who will undergo a right subclavian vein doppler study

1. administering Nitro is within the LPN/LVN practice

The nurse is working with an experienced unlicensed assistive personal and an LPN/LVN on a tele unit. A pt who has an acute MI 3 days ago has been reporting fatigue and chest discomfort when ambulating. Which nursing activity included in the care plan best assigned to the LPN/LVN? 1. administering nitroglycerin 0.4mg sublingually if chest discomfort occurs during client activities 2.monitoring pulse, BP, and o2 before and after ambulation 3. teaching the pt energy conservation techniques to decrease myocardial oxygen demand 4.explaining the rationale for alternating rest periods with exercise to pt and family.

3. Baked, broiled, or grilled meats are recommended; a plain baked potato is appropriate; and skim milk is low in fat—so this meal is appropriate for a low-fat, low-cholesterol diet. 1. Fried foods are high in fat and cholesterol. 2. White bread is not high in fiber; wheat bread should be recommended because it is high in fiber. Whole milk is high in fat; skim milk should be used. 4. Hamburger meat is high in fat, French fries are usually cooked in oil (which is high in fat), and carbonated beverages are high in calories.

The nurse knows the client understands the teaching concerning a low-fat, low- cholesterol diet when the client selects which meal? 1. Fried fish, garlic mashed potatoes, and iced tea. 2. Ham and cheese on white bread and whole milk. 3. Baked chicken, baked potato, and skim milk. 4. A hamburger, French fries, and carbonated beverage.

3. possible AE of ACE inhibitors

The nurse makes a home visit to evaluate a HTN pt who has been taking enalapril. Which finding is most important to report to the HCP? 1. pt reports frequent urination 2. BP 133/86 3.complains of dry cough 4."i get dizzy when i stand up fast"

1. An S3 heart sound indicates left ventric- ular failure, and the nurse must assess this client first because it is an emergency situation. 2. The nurse would expect a client with CHF to have sacral edema of 4+; the client with an S3 would be in a more life-threatening situation. 3. A pulse oximeter reading of greater than 93% is considered normal. 4. An elevated creatinine level is expected in a client diagnosed with chronic renal failure.

The nurse on the telemetry unit has just received the a.m. shift report. Which client should the nurse assess first? 1. The client diagnosed with myocardial infarction who has an audible S3 heart sound. 2. The client diagnosed with congestive heart failure who has 4+ sacral pitting edema. 3. The client diagnosed with pneumonia who has a pulse oximeter reading of 94%. 4. The client with chronic renal failure who has an elevated creatinine level.

C. still swollen so it might look worse before it looks better

The nurse plans care for a pt diagnosed with thrombotic stroke with the knowledge that over the next 72 hours he: a. will be ready for aggressive rehab b. will show gradual improvement of the initial neurologic deficits c. may show signs of deteriorating neurologic function as cerebral edema increases d. should not be turned or exercised to prevent extension of the thrombus and increased neurologic deficits

B. older people have this issue when they have cardiac ishcmeia, how old people present

The older patient with coronary artery disease (CAD) is more likely to have what symptom if experiencing cardiac ischemia? a. syncope b. dyspnea c. chest pain d. depressio

B since the pt can swallow

The patient admitted with a right-sided stroke needs assistance with feeding, but can swallow well. To whom would it be best to delegate this responsibility? A.Licensed practical nurse B.Certified nursing assistant C.Hospital volunteer D.Student nurse doing first patient care experience

No, 3-4.5 hours , more risk for bleeding

The patient is admitted to the acute medical unit after 7 hours. His wife asks if her husband will receive IV thrombolytic therapy. What is your best response?

C

The spouse of a patient brought to the ED states that 6 hours ago her husband began having difficulty finding words. The patient has since become progressively worse. He has right hemiparesis. Upon assessing the patient, you note that he is lying flat in a supine position, is drooling, and has been incontinent of urine.Place in order of priority the interventions the nurse should take for this patient at this time? A.Provide perineal care B.Assess for gag reflex C.Elevate the head of bed D.Perform a linen and gown change

B, best option

The wife of a patient admitted with a left-sided stroke must leave her husband's bedside for 2 hours to run errands. Which nursing action is appropriate to contribute to patient safety while she is gone? A.Apply restraints to the patient's wrists. B.Maintain the bed in a low position. C.Sit with the patient until his wife returns. D.Place the call light in the patient's right

No ,has to get gag test

Thirty minutes later, the wife asks for a glass of water or juice because her husband is thirsty. What is your best response?


Kaugnay na mga set ng pag-aaral

WGU C207 Data Driven Decision Making Module 4

View Set

Biology: CH 1, Biology: CH 2, Biology: CH 3, Biology: CH 4, Biology: CH 5, Biology: CH 7, Biology: CH 8, Biology: CH 9, Biology: Ch 10, Biology: CH 11

View Set

Junior English 2nd Semester : 5. REVIEWING COMMUNICATION SKILLS AND LITERATURE

View Set

Psychology 10,1 Chapter 11: Personality

View Set

Midterm 6,7,8,10 Extra Review Questions

View Set

Macroeconomics: Chapter 6 Launchpad

View Set